MCQs and Cases 5th Year

You might also like

Download as docx, pdf, or txt
Download as docx, pdf, or txt
You are on page 1of 118

MCQs and Cases

In medicine

For 5th year medical students

Dr . Arwa kara

2022

1
Renal MCQs
in patient with acute renal failure and red cell cast in the urine, which .1
? of the following diagnosis should be considered
A. acute tubular necrosis
B. obstruction
C. rhabdomyolysis
D. acute glomerulonephritis
E. acute interstitial nephritis
recognized feature of adult polycystic kidney disease, include which .2
one of the following
A. small kidneys on ultrasound scan is diagnostic
B. normal blood pressure is characteristic

2
C. there is significant erythropoietin deficiency
D. autosomal dominant inherited disease
E. nephrolithiasis and infection is very rare
which of the following is an indication of hemodialysis .3
A. fluid over load responsive to diuretics
B. hypokalemia
C. severe metabolic acidosis
D. decrease in urine out put
E. hypernatremia
which of the following best describes patient with chronic kidney .4
disease
A. progressive azotemia over weeks
B. normal size both kidney on ultrasound scan
C. oliguria in dehydrated patient
.D. estimated GFR of less than 60ml /min
which of the following conditions of chronic kidney disease is .5
associated with small sized kidney on USS
A. gouty nephropathy
B. multiple myeloma
C. amyloidosis
D. diabetes mellitus
in patient with renal impairment anti glomerular basement membrane .6
antibody is highly suggestive of
A. IgA nephropathy
B. polyarterities nodosa
C. sarcoidosis
D. good pasture syndrome
pre renal azotemia may be due to .7
A. immune complex glomerulonephritis
B. vasculitis
C. aminoglycoside toxicity

3
D. myocardial infarction
the following are true regarding minimal change nephropathy except .8
A. is the commonest cause of nephrotic syndrome in adult
B may be associated with Hodgkin disease
C. no immune complex deposit in histopathology
D. good response to steroid
the following cause membranous nephropathy except .9
A. HBV
B. Gold therapy
C. bronchogenic carcinoma
D. acute rheumatic fever
E. malaria
all the following are recognized feature of CRF except .10
A. nausea
B. pruritus
C. polycythemia
D. hypocalcemia
E. hyperphosphatemia
which of the following is a type of large vessels vasculitis .11
A. poly-arteritis nodosa
B. Churg-strauss syndrome
C. Takayasu's arteritis
D. Kawasaki disease
E. Henoch-Schonlein purpura
12. which of the following is considered as non-autoimmune
glomerulopathy
A. lupus nephritis
B. H-S purpura
C. post-streptococcal glomerulonephritis
D. Good pasture's syndrome
E. Diabetic nephropathy

4
13. which of the following investigations is most useful in diagnosis of
acute glomerulonephritis
A. serum creatinine
B. Serum calcium
C. serum phosphorus
D. blood urea nitrogen
E. urine for RBCs cast
14. recognized feature of adult polycystic kidney disease include
A. autosomal recessive inherited disease
B. protein urea is common at presentation
C. male individuals are priority for family screening
D. associated with intracranial aneurysm
E. patients are frequently anemic by the time they develop renal failure
15. which of the following condition is an indication for immediate
dialysis
A. PH more than 7.2
B. hyperkalemia not responding to medical treatment
C. uremic gastritis
D. decrease in urine output
E. hypercalcemia
16. which one of the following is true for patient with chronic kidney
disease
A. hypokalemia usually develops when GFR falls to less than 20-25ml/min
B. patient on chronic dialysis have lower incidence of morbidity and mortality
C. the commonest cause of death in dialysis population is cardiovascular
disease
D. metabolic alkalosis is recognized feature
E. patient with end stage renal disease who undergo renal transplantation
survive shorter than those on chronic dialysis
17. which of the following is recognized cause of hypernatremia
A. hypothyroidism
B. SIADH

5
C. adrenal insufficiency
D. diabetes insipidus
E. volume overload
18. the likely cause of hyponatremia with increased urinary sodium
excretion
A. diabetes insipidus
B. nephrotic syndrome
C. vomiting
D. adrenal insufficiency
E. paraproteinemia
19. regarding hyponatremia which of the following is true
A. diabetes insipidus is a recognized cause
B. cerebral edema occurs when serum sodium drops to 130 mmol/l
C. in SIADH hyponatremia results from intracellular volume depletion
D. hypotonic saline is treatment of choice for severe hyponatremia
E. rapid correction of hyponatremia may cause central pontine myelinolysis
20. which of the following is the commonest cause of end stage renal
disease
A. polycystic kidney disease
B. hypertension
C. diabetes mellitus
D. reflux nephropathy
E. Reno vascular disease
21. anemia of chronic disease is primarily due to
A. erythropoietin deficiency
B. hemolysis
C. vitamin B12 deficiency
D. hematemesis
E. reduced synthesis of active vitamin D
22. all of the following are recognized complications of chronic renal
failure except

6
A. normocytic normochromic anemia
B. osteomalacia
C. renal cell carcinoma
D. pericarditis
E. peripheral neuropathy
23. which of the following is not a complication of chronic renal failure
A. infections
B. hypocalcemia
C. acidosis
D. hypophosphatemia
E. bleeding
24. the following are causes of high anion gap metabolic acidosis except
A. DKA
B. acute and chronic renal failure
C. aspirin intoxication
D. acetazolamide therapy
E. alcohol intoxication
25. indication of dialysis in CRF include the following except
A. uremic symptoms
B. pericarditis
C. pulmonary edema
D. hypocalcemia
E. hyperkalemia
26. in adult polycystic kidney disease which is true
A. inherited as AR disorder
B. malignant changes is common
C. SAH is recognized complication
D. all patient have hepatic cyst
E. most patient presents with anemia
All the following indicates dialysis in patients with end stage renal .27
disease except

7
A. Pulmonary edema
B. Uremic encephalopathy.
C. Blood Ph of 7.28
D. Uremic pericarditis.
E. Hyperkalemia.
28. All are true in acute renal failure except
A. high urea
B. high hydrogen concentration
C. hypercalcemia
D. hyperkalemia
Causes of increased anion gap metabolic acidosis include all of the .29
following except
A. Diabetic ketoacidosis.
B. Lactic acidosis.
C. Renal tubular acidosis.
D. Methanol poisoning.
E. Renal failure.
30. The followings are recognized features of IgA nephropathy except:
A. Hematuria.
B. Hypertension.
C. Presented 2 - 3 weeks after minor respiratory infection.
D. Serum complement levels are normal.
E. A common cause of end stage renal disease ESRD.
31. Hypokalemia may be caused by all of the followings except:
A. Angiotensin converting enzyme inhibitor therapy.
B. Thiazide diuretic therapy.
C. Loop diuretic therapy.
D. Primary hyperaldosteronism.
E. Secondary hyperaldosteronism.
32. HB-s-Ag is present in Vasculitis associated with :
A. Henoch-schonlein purpura

8
B. Temporal arteritis
C. Churg- Struss syndrome
D. Polyarteritis nodosa (PAN )
E. Wegner granulomatosis
33. In Churg-Struss syndrome, the principle organ involved is :
A. Kidney
B. Lung
C. Central Nervous system
D. Liver
E. a and b are involved

Renal Cases
1. A 36-year-old hypertensive man develops macroscopic hematuria 24hour
after onset of pharyngitis. The patient brother had history of post streptococcal
glomerulonephritis at age 6 years.
What is the most likely explanation of this patient hematuria?
A. glomerulosclerosis
B. IgA nephropathy
C. post streptococcal glomerulonephritis
D. renal vein thrombosis
2. 68 year old man , who presented with increasing headache and dizziness
for three days. O/E BP 188/118 mmHg. With weak peripheral pulses, bilateral

9
carotid bruit, no pedal edema .laboratory data : urine mild proteinuria, no RBC
cast, urea 70mg/dl, creatinine 1.8mg/dl
Which of the following is most likely cause for his severe headache?
A. Coarctation of aorta
B. chronic kidney disease
C. Cushing syndrome
D. acute glomerulonephritis
3. A 70-year-old male diagnosed with knee osteoarthritis and has been
consuming large quantities of diclofenac sodium for the last 5 years because
of the pain, presents with hypertension and renal impairment.
Laboratory investigation: blood urea nitrogen 150 mg/dl, serum creatinine
3.2mg/dl , eosinophilia
Which is the most likely diagnosis?
A. Wagner's granulomatosis
B. good pasture syndrome
C. chronic interstitial nephritis
D. mesangiocapillary nephropathy
4. A 24 year old female patient presented to nephrology OPD with decreased
level of consciousness. Urgent investigations were done showed : RFT urea
301mg/dI ( N˂40 mg/dl) S. creatinine = 10 meq /L (N 0.6 -1.2 meq/L),Na-135
meq/L (N 135 -145 meq/L) . K= 5.1 meq/L (N 3.5 -5 meq/L)
What is the best therapeutic approach at this moment
A. I.V fluid.
B. IV Dextrose 50% and 1Ounits Regular insulin
C. Family consent for urgent hemodialysis.
D. I.V calcium gluconate 10% in 10cc Normal saline.
F. I.V frusemide
5. A 10-year-old male patient presented with history of progressive dyspnea,
oliguria, and smoky colored urine of 3 days duration. He gave history of upper
respiratory tract infection 2 weeks before onset of his illness. O/E BP
190/110mmHg. Investigation: BUIN 168mg/dl, creatinine 4.5mg/dl ,urine
analysis proteinuria +++, RBCs 20-30 HPF with casts
The most likely diagnosis is
A. IgA nephropathy

10
B. minimal change disease
C. post streptococcal glomerulonephritis
D .hypertension with nephrosclerosis
6. middle aged women known case of chronic glomerulonephritis. Her last
estimated GFR was found 28ml/min. she presented with progressive fatigue
and palpitation, she is on Lisinopril 10 mg twice daily for her hypertension, no
pallor, no leg swelling
Which of the following biochemical investigation is priority in this
patient
A. serum potassium
B. serum ferritin
C. serum phosphorus
D. serum sodium
E. serum calcium
7. A 68 year-old male with chronic renal insufficiency. presents with
weakness, and paresthesia. laboratory finding shows potassium level of : 7.2
meq/L. ECG : peaked T wave
Which one of the following is first line immediate treatment
A. Intravenous calcium and intravenous glucose and insulin
B. Dialysis.
C. Sodium polystyrene sulfonate powder.
D. Intravenous magnesium infusion
E. Ameloride diuretic
8. A 45 year-old male is seen in the emergency department with nausea and
lethargy. Past medical history unremarkable. Blood sample shows the
following: Na 140 mmol/l, K 6.8 mmol/l. serum creatinine 2.6 mg/dl, urea 110
mg/dl
What is the most appropriate initial management
A. hemodialysis
B. insulin an dextrose infusion
C. nebulized salbutamol
D. I/V calcium gluconate
9. previously healthy young teacher, presented with history of bilateral joint
pain, arthralgia and macular skin rash over legs and abdomen for three days.

11
No previous history of sore throat or dysentery. She used diclofenac sodium
tablet for her dysmenorrhea a week ago
Which of the following is likely to be found in this patient laboratory data
A. granular cast
B. urate crystals on urine examination
C. high ASO titer
D. eosinophilia
E. nephrotic range protein urea
10. A 54 year-old female with diabetic nephropathy comes to the office
because she had vomiting, anorexia, fatigue, itching and a metallic taste in
her mouth for the past 2 weeks. RR 32/min. physical examination shows urine
like odor of the breath and pitting edema. Estimated GFR 14ml/min
Additional studies are most likely to show presence of the following
condition
A. primary metabolic alkalosis
B. primary metabolic acidosis
C. primary respiratory alkalosis
D. primary respiratory acidosis
11. A 50 year-old male is hospitalized for acute MI. he has decreased cardiac
output with hypotension and requiring multiple vasoconstrictor agent . his
urine output drops over the next three days. Serum urea nitrogen increased to
99mg/dl with creatinine 2.7 mg/dl. Urine analysis reveals no protein, glucose
or blood. Numerous hyaline casts. Five days later he develops polyuria and
serum urea nitrogen declines toward normal
Which of the following pathological finding most likely has caused his
azotemia
A. crescentic glomerulonephritis
B. papillary necrosis
C. acute tubular necrosis
D. focal segmental glomerulonephritis
12. A 46 year-old female develops nephrotic syndrome and is awaiting further
test to establish the underlying etiology
In which circumstance would corticosteroids be most effective in
reversing the nephrotic syndrome
A. renal artery thrombosis

12
B. primary amyloidosis
C. minimal change disease
D. membranous nephropathy
13. A 30 year old male has noted puffiness around his eyes and swelling of
his feet for the past 2 weeks. On examination his BP 140/80 mmHg. Urine
microscopic examination reveals oval fat bodies
Which of the following condition is he most likely to have
A. nephritic syndrome
B. papillary necrosis
C. obstructive uropathy
D. nephrotic syndrome
E. ascending pyelonephritis
14. A 56 year-old male referred to renal clinic with history of bilateral leg
swelling, history of mild exertional dyspnea and cough but no orthopnea. His
serum albumin is 2.3g/dl (normal 4-6), urine routine by dipstick is +++, urine
RBCs is 4-6 HPF, while urine WBCs is normal and no urinary sediment casts
Upon further workup, which of the following is likely to be present
A. urine granular cast
B. hypercholesterolemia
C. macroscopic hematuria
D. hypernatremia
15. A 49 year-old female has been hospitalized for the past 10 days for
treatment of bronchopneumonia. She has developed chills and fever over the
past 2 days. On examination: temperature 35.5C and she has diffuse
erythematous skin rash. Lab results show serum creatinine 2.2mg/dl, glucose
73mg/dl and peripheral blood film shows eosinophilia. On urine analysis, she
has ++ proteinuria but no blood, glucose or ketones
The most likely diagnosis is
A. IgA nephropathy
B. acute tubular necrosis
C. post streptococcal glomerulonephritis
D. drug induced interstitial nephritis
16.A 44-year-old female presents with recurrent sinusitis. She has failed to
respond to multiple antibiotic courses and has recently developed hemoptysis
dyspnea and pleuritic chest pain., On physical examination, temperature is

13
36.8 C blood pressure is 160/100 mmHg, pulse rate is 92/min and respiratory
rate is 18/min Arterial oxygen saturation is 98% on ambient air. Her nose Is
saddle shaped she has no evidence of synovitis, rash, or mononeuritis,
pulmonary examinations shows dullness to percussion at right middle and
lower zone with bronchial breathing and crackles.
What is the most likely diagnosis
A. Bechet's disease.
B. Good pastures' syndrome.
C. Massive pulmonary embolism.
D. Systemic lupus erythematosus.
E. Wegener granulomatosis.
17. A 20 years old African male patient, who has presented with recurrent
abdominal ischemic crisis on background of chronic hemolytic anemia
Which of the following would be expected to find
A. Decreased urinary concentrating ability.
B. Myoglobiunuria
C. Nephrolithiasis.
D. Hydro nephrosis.
E. Nephrotic range proteinuria
18. A 62 years old man with severe aortic regurgitation waiting for aortic valve
replacement surgery, presented with increasing dyspnea and signs of fluid
retention, BP 118/54 mmHg ,gallop rhythm, and end inspiratory crackles. He
has been on medical therapy including furosemide 40 twice per day,
bisoprolol 5mg once per day. His cardiologist sent for biochemical workup
which showed the following: Na 134 mEqL, K 3.8 mEq/L, Cl 98 mEq/L, urea
42 mg/dL, creatinine 1.2 mg/dL, arterial blood gas showed: pH 7.52 (7.3-7.45)
PC0248mmHg (40mmHg) HCO334 mEq L(24 mEq/L).
Which of the following acid-base disorders this patient has developed
A. Metabolic alkalosis.
B. Respiratory alkalosis,
C. Renal tubular acidosis.
D. Diuretic induced metabolic acidosis.
E. Respiratory acidosis.
19. A urine examination of 54-year old patient showed: Protein ++ WBCs
4/hpf (N <5) RBCs /Casts positive .Bacteria negative

14
This is compatible with:
A. Renal stones.
B. Glomerulonephritis.
C. Pyelonephritis.
D. Prostatitis,
B. Post cystoscopy.
20.The following is a result of an arterial blood gas; Pa02 90 mmHg(N 75-
100mmHg) PaCO2 39 mmHg(N 35-45) pH 7.21(N 7.35-7.45) ,Bicarbonate 12
mEq/L (N 22-26). K+ =5.3 mEq/L Na+ 149 mEq/L Cl-= 90 mEq/L
These data are compatible with:
A. metabolic alkalosis
B. High anion gap metabolic acidosis.
C. Fully compensated metabolic acidosis.
D. Normal anion gap metabolic acidosis.
E. Respiratory acidosis compensated with respiratory alkalosis.

Endocrine MCQs
1. the most common cause of primary hypothyroidism is
A. Sheehan syndrome
B. post-partum thyroiditis
C. post radioactive iodine ablation
D. post thyroidectomy
E. Hashimoto thyroiditis
2. which one of the following is associated with increased risk of
osteoporosis
A. increased body mass index
B. male gender

15
C. early menarche
D. untreated hypothyroidism
E. use of corticosteroid
3.which one of the following indicates vitamin D deficiency
A. low calcium, high phosphorus and low parathyroid hormone
B. Albright osteodystrophy, low calcium, high phosphorus and high
parathyroid hormone
C. low calcium, low phosphorus and high parathyroid hormone
D. Albright osteodystrophy, normal calcium, normal phosphorus and normal
parathyroid hormone
E. high calcium, low phosphorus and high parathyroid hormone
4. regarding primary hyperaldosteronism which one of the following is
true
A. adrenal carcinoma accounts for more than 60% of cases
B. it is mainly caused by hyper reninemia
C. hypertension and hypokalemia is well recognized feature
D. spironolactone is contraindicated
E. central diabetes insipidus is characteristic feature
5.regarding primary hyperparathyroidism which one of the following is
true
A. it is usually a complication of chronic renal failure
B. polyuria is a recognized feature and is caused by nephrogenic diabetes
insipidus
C. a 24 hour urine calcium excretion is usually low
D. after parathyroidectomy rebound hypercalcemia is a recognized
complication
6.time for screening for diabetic retinopathy in a patient with type 2
diabetes is
A. after 3 years
B. after 10 years
C. at time of diagnosis
D. only if there is nephropathy

16
7. which one of the following anti diabetic medications acts by
increasing insulin secretion
A. biguanides ( metformin)
B. SGLUT-2 inhibitors
C. alpha glucosidase inhibitors
D. glucagon like peptide-1(GLP-1) agonist
E. thiazolidinedione
8.which one of the following suggests a diagnosis of primary adrenal
insufficiency ( Addison disease)
A. hypertension
B. hyperglycemia
C. weight gain
D. hyperpigmentation
9.which one of the following oral antidiabetic drugs act mainly by
increasing insulin sensitivity
A. DPP4 inhibitors
B. GLP-1 agonist
C. thiazolidinedione
D. SGLUT-2 inhibitors
10.which one of the following indicates hypoparathyroidism
A.↓ calcium, ↑ phosphorus, ↓ parathyroid hormone
B. ↓ calcium, ↓ phosphorus, ↑ parathyroid hormone
C. ↑ calcium, ↓ phosphorus, ↑ parathyroid hormone
D. Albright osteodystrophy, ↓calcium, ↑ phosphorus,↑ parathyroid hormone

11. regarding osteoporosis, which of the following is true


A. estrogen increase the progression of osteoporosis
B. bed rest and decrease physical activity are recommended to prevent
fracture
C. back pain is early feature of osteoporosis
D. corticosteroid is recognized cause of osteoporosis

17
E. WHO definition of osteoporosis is based on plain x-ray feature of spine,
wrist and hip joint
12.regarding primary hyperparathyroidism, which of the following is true
A. low serum vitamin D support the diagnosis of primary hyperparathyroidism
B. 90% of cases caused by parathyroid carcinoma
C. urinary calcium is low in primary hyperparathyroidism
D. familial type of primary hyperparathyroidism associated with MEN type 1
and 2
13.the most common cause of endogenous Cushing is
A. adrenocortical carcinoma
B. adrenal adenoma
C. ACTH secreting pituitary adenoma
D. ectopic ACTH secreting tumor
14. vitamin D deficiency is best diagnosed by measuring serum level of
A. total calcium
B. alkaline phosphatase
C. 25 hydroxyvitamin D
D. 1-25 hydroxyvitamin D
15. clinical feature of acromegaly include all the following except
A. goiter and cardiomyopathy
B. hypertension and impaired glucose tolerance
C. increase sweating and headaches
D. skin atrophy and decreased sebum secretion
16. regarding pituitary adenoma all are true except
A. recommended screening test in patient suspected GH secreting adenoma
is IGF-1
B. prolactin secreting tumor is the most common tumor
C. central diabetes insipidus is a common finding at presentation
D. ACTH secreting tumor more common in female
E. hypopituitarism is a recognized presentation
17 .in solitary thyroid nodule, the following are associated with
increased risk of malignancy except

18
A. age more than 70 years at presentation
B. family history of thyroid cancer
C. female
D. history of head and neck radiation
E. micocalcification in USS
18.regarding dyslipidemia which one of the following is true
A. level of dyslipidemia more important than patient risk profile
B. familial hypertriglyceridemia is an autosomal recessive condition
C. hypothyroidism is associated with hypercholesterolemia
D. myalgia and myositis are common side effects of treatment with fish oil
19.one of the following is not feature of hypothyroid( myxedema coma(
A. hypothermia
B. hypernatremia
C. hyponatremia
D. hypoglycemia
E. hypoxemia
20.in pheochromocytoma , which of the following is most common
symptom
A. sustained hypertension
B. paroxysmal hypertension
C. orthostatic hypertension
21.The following causes hypoglycemia, EXCEPT:
A. Addison's disease
B. Glucagonoma
C. Uremia
D. Hepatocellular failure
22. The following are laboratory findings in patients with osteomalacia ,
EXCEPT
A. Low calcium
B. Low phosphate
C. High alkaline phosphatase

19
D. Low PTH level
E. Low 25-OH vitamin D level
23. Polyuria is a feature of all the following, EXCEPT:
A. Hypercalcemia
B. Hypokalemia
C. Hyperglycemia
D. Nephrogenic diabetes insipidus
E. Inappropriate ADH secretion
24.The followings are features of Acromegaly, EXCEPT
A. Arthropathy
B. Hypertension
C. Carpal tunnel syndrome
D. Hypoglycemia
25. The following are suggestive of Addison's disease, except:
A. Hyponatremia.
B. Loss of axillary hair.
C. Postural hypotension.
D. Palmar pigmentation.
E. Hypokalemia.
26. Which one of the following is included in the features of multiple
endocrine neoplasia MEN type 1
A. Adrenal nodule.
B. Hyperparathyroidism.
C. Hyperthyroidism.
D. Hypothyroidism.
E. Phaeochromocytoma.
27. Which of the following tests are used to detect and monitor
osteoporosis
A. Dual- photon absorptiometry.
B.Dual- energy X-ray absorptiometry.
C. Radiography.

20
D. Quantitative CT scan.
E. Single photon absorptiometry.
28. The finding of reduced serum free T4 and thyroid-stimulating
hormone TSH concentrations is compatible with the following
conditions :
a. Primary hypothyroidism
b. Subclinical hypothyroidism
c. Hypopituitarism
d. Pneumonia
e Both c and d

Endocrine cases
1. A 50-year old man previously healthy, presented to emergency department
with random blood glucose of 250 mg/dl, he had history of polyuria and
polydipsia for 1 month
What should be the next most appropriate step?
A. repeat blood glucose after 12 hour fasting
B. he is confirmed to have diabetes and should be referred to diabetic clinic
for diabetes management
C. he has to do a 75g oral glucose tolerance test
D. start sulphonyl urea
E. start DPP4 inhibitors
2. A 38-year old woman presented with newly diagnosed diabetes, weight
gain and hirsutism. She had oligo menorrhea. By examination she had round

21
face, central obesity, purple wide abdominal striae and acne. Her blood
pressure 180/90 mmHg.
What is the most likely diagnosis?
A. polycystic ovary syndrome
B. Cushing syndrome
C. Conn's syndrome
D. acromegaly
E. metabolic syndrome
3. A 60-year old female presented with lethargy and weight loss. She gave
history of increasing skin pigmentation. By examination she was thin with dark
skin and buccal pigmentation. Blood pressure 100/60mmHg. Investigation
revealed serum sodium 133 mmol/l (N= 135-145). Potassium 4.7 mmol/l (N=
3.5-4.5). blood glucose 60 mg/dl. Serum calcium 10.8 mg/dl ( N=8.5-10.4)
What is the most appropriate investigation you would do next?
A. short synacthen test
B. serum parathyroid hormone
C. 1 mg overnight dexamethasone suppression test
D. oral glucose tolerance test
E. serum 25 hydroxy vitamin D
4.A 31 year old single lady was referred for evaluation for secondary
amenorrhea associated with galactorrhea and headache. Past and drug
history were unremarkable. Renal, liver and thyroid function test were normal
Which of the following should be the initial diagnostic test?
A. CT scan of the pituitary
B. measurement of serum prolactin
C. measurement of plasma IGF-1
D. insulin tolerance test
5. A 28-year old woman presented with history of paroxysmal attacks of
headache , palpitation and sweating. On examination she looks anxious and
pale, her blood pressure is 200/120mmHg, pulse rate 110 b/min. laboratory
investigations serum potassium 4.4 mmol/l. serum sodium 136 mmol/l fasting
blood glucose 170mg/dl.
What is the most appropriate initial diagnostic test?
A. CT scan of the abdomen

22
B. plasma catecholamine
C. 24 urine for vanillyl mandalic acid VMA
D. ultrasound of the abdomen
6. A 56-year old woman with type 2 diabetes mellitus for 18 years,
complaining intolerance to daily activities with bone aches. Her creatinine
clearance found to be 28ml/min./1.73m².
Which one of the following abnormalities is likely to be seen in this
patient?
A. hypercalcemia
B. hypophosphatemia
C. elevated parathyroid hormone
D. hypokalemia
7. A 50 years old male patient presented with headaches, palpitations and
sweating. On examination: he is anxious, irritable, heart rate 140bpm, blood
pressure 220/120. ECG sinus tachycardia. Blood sugar 180, hemoglobin
15g/dl.
What is the most appropriate investigation to confirm the diagnosis
A. thyroid function test
B. serum Vinyllmandelic acid(VMA)
C. 24 hour urine collection for metanephrines and normetanephrines
D. serum insulin and C peptide level
E. serum 5-hydroxyindoleacetic acid (5-HIAA)
8. A 40 year old male presented with nocturia , fatigue and muscle aches.
Investigations: FBS 140mgldl, HBA1C 6.8%, and CBC, RFT, LFT all are
normal
What is the most appropriate next step
A. repeat fasting blood glucose after 6 months
B. reassure the patient as his investigation are normal
C. order oral glucose tolerance test
D. he had prediabetes and needs annual blood glucose measurement
E. start life style modifications and antidiabetic medications
9. A 35 year old female complains of generalized fatigue, constipation, and
weight gain. By examination, she is pale with puffy face. She had dry skin. PR
55/min regular, BP 130/90 mmHg. She had delayed relaxation of ankle jerk

23
Which of the following investigations is recommended to diagnose this
condition
A. FT3 and TSH
B. FT4 and TSH
C. total T4 and total T3
D. TSH receptor antibody
E. free T4 index
10. A 23 year old women presented with 6 month history of anorexia, mild
abdominal pain, weight loss and oligo menorrhea, she had vitiligo diagnosed
5 year earlier. On physical examination: her BMI 18.5kg/m2, BP 96/54 mmHg.
She had generalized skin hyperpigmentation most marked at knuckles and
buccal mucosa and the tongue
The most likely diagnosis is
A. celiac disease
B. Crohn's disease
C. Conn's disease
D. irritable bowel syndrome
E. Addison's disease
11. Which of the following is the most appropriate initial test is
A. short ACTH test
B. overnight mettyrapone test
C. plasma renin activity
D. insulin tolerance test
12. A 50 year old female presented with generalized fatigue, dry skin,
sleepiness and constipation for several months. On examination: she looks
apathic, has loss of scalp hair, periorbital puffiness and small goiter.
PR58/min, BP 130/100 mmHg. Investigations: TSH 60 (normal 0.3-4).FT4 6
(normal 10.5-20). Anti-thyroid peroxidase antibody positive
The most likely diagnosis is
A. Hashimoto's thyroiditis
B. Grace's disease
C. Sheehan syndrome
D. TSH secreting pituitary adenoma
E. De Quervain thyroiditis
24
13. A 40 year old female nurse complains of menstrual irregularities in the few
months. She is ion treatment for bronchial asthma for the last 10 years.
Physical examinations revealed obesity, some pink abdominal striae and mild
hypertension
The most appropriate assessment to rule out Cushing syndrome in this
women
A. CT scan pituitary gland
B. overnight 1mg dexamethasone suppression test
C. 2 day 8mg/day dexamethasone suppression test
D. measuring of morning plasma ACTH level
E. MRI pituitary gland
14. A 27 year old male presented with headaches ,polyuria and polydipsia for
one month back. He denies any weight loss or any medications. Examinations
was normal, investigations: serum Na 148( normal 135-145), serum K 3.9
(normal 3.5-4.5), FBG 87mg/dl
What is the most appropriate next step
A. perform 75g oral glucose tolerance test
B. water deprivation test
C. venous blood gases
D. MRI brain
15. A 70 year old female admitted to medical ICU with pneumonia and
hypotension. Clinically she looks under- weight. PR 56/min, temperature 37C,
blood test show hyponatremia, hyperkalemia and low TSH. She received IV
fluids and antibiotics. However, her blood pressure is still low
Which of the following is the most appropriate action
A, short ACTH stimulation test
B. hydrocortisone followed by thyroxine
C. urgent MRI brain
D. insulin stress test
16. A 47 year old female presented with headaches, abdominal distension,
and muscle weakness. Physical examination is normal except BP
180/110mmHg. Initial investigation shows: serum K 3.2(normal 3.5-5), serum
Na 145(normal 135-145), serum urea 20mg/dl, serum creatinine 1.2 mg/dl. On
urine analysis she has proteinuria. Further evaluation revealed an elevated
plasma aldosterone concentration/plasma renin ratio
The most likely diagnosis is

25
A. Barter's syndrome
B. Conn's syndrome
C. Liddle syndrome
D. Renal secreting tumor
E. renal artery stenosis
17. A 32 year old women present with a complaint of poor caesarean section
wound healing and failure of lactation. The patient delivered a full term
neonate 2 weeks ago. Her delivery was complicated by maternal hemorrhage
and severe hypotension
What is the most likely diagnosis
A. hypothyroidism
B. toxic shock syndrome
C. improper lactation technique
D. Sheehan's syndrome
18. A 10 years old boy is brought I by his mother with abdominal pain,
polyuria and polydipsia. Diagnosed with severe DKA. And appropriate
treatment is initiated, after 6 hours the patient develop severe headaches,
vomiting and depressed mental status
What is the likely diagnosis
A. hyperosmolar hyperglycemic state
B. hyperglycemic seizure
C. cerebral edema
D. migraine headaches
19. A 51-year-old obese woman presented to the Clinic for review of her
investigations. She had a history of hypertension and dyslipidemia, which is
managed with Ramipril and atorvastatin. On examination, her body mass
index (BMI) is 33 kg/m, and her blood pressure (BP) is 120/70 mmHg .Her 8
hours fasting blood glucose was 128 mg/dl, her HBA1C was 6.8%.
Which of the following is the most appropriate next step in her
management
A. Follow up after one year and no treatment is needed
B. Refer her to diabetic clinic to start treatment
C. She only had pre- diabetes.
D. Her investigations are normal and she needs reassurance.

26
E. Refer for glucose tolerance test after 75 g oral glucose load.
20. A 23-year-old woman presents to the Clinic. She has had type I diabetes
for eight years, and until recently it has been well controlled; however,
recently, She has been having numerous hypoglycemic attacks. These
'hypos' can be at any time of the day and follow no pattern. Her periods are
regular. On examination, her blood pressure (BP) is 100/60 mmHg with a
postural drop of 10 mmHg. She is on a basal-bolus regime, taking insulin
eight units three times per day and glargine insulin 15 units at night.
Investigations reveal the following: Hemoglobin (Hb) 10 g/dl (11.5-15.5 ),
Sodium (Na) 130 mmol/1 135-145 mmo1/1, Potassium (K) 5.4 mmol/L
(normal value 3.5-5.0 mmol/I), Creatinine (Cr) 1 mg/dI (normal value 0.5- 1.2),
Mean Corpuscular Volume (MCV) 93 fl (N 76-96 fl) Urea 50 mg/d1 (normal
value 20-40)
Which of the following is most likely to determine the underlying
diagnosis
A. Serum ferritin.
B. Paired serum and urine osmolality
C. Continuous glucose monitoring
D. Short Synacthen test.
E. Transferrin saturation
21. A 34-year-old woman presents with weight loss, palpitations and anxiety.
Her periods stopped three months ago, but a pregnancy test which she took
two weeks ago was negative. On examination, she has a blood pressure (BP)
of 150/70 mmHg, her pulse is 110/min at rest and her body mass index (BMI)
is 21. She has an Investigations reveal the following: Obvious diffuse goiter
with bruit. Hemoglobin (Hb) 12.1 g/dl (normal value11.5-15.5 g/dl)
Thyroid-stimulating hormone (TSH) <0.05 mU/I) ( normal value 0.3-3.2 mU/l)
What is the most likely diagnosis
A. Toxic Graves' disease.
B. Hashimoto's thyrotoxicosis
C. Toxic adenoma
D. Toxic multinodular goiter.
E. Subacute thyroiditis.
22. A 56-year-old man with a history of hypothyroidism managed with
thyroxine replacement comes to the clinic for review. He has increasing
tiredness and facial numbness. Investigations: Thyroid stimulating hormone
(TSFI) 2.1 mU/l ( normal value 0,3-3.2 u/I) Hemoglobin 11.3 g/d1 ( normal

27
value 13.5-17.5 g/dI) Sodium (Na) 139 mmol/l (normal value 135-145 mmol/l)
Potassium (K) 5.4 mmoI/l (normal value 3.5-5.0 mmol/I) Creatinine 0.7 mg/dl
(normal value 0.2- 1.2) Corrected Calcium 6.5 mg/dI (normal value 8.5- 10.5)
Parathyroid hormone (PTH) 13 mmol/l (normal value 1-6 mmol/I)
What is the most likely diagnosis
A. Hypoparathyroidism
B. Celiac disease.
C. Pseudo- pseudo hypoparathyroidism.
D. Vitamin D deficiency.
E. Primary hyperparathyroidism.

Rheumatology MCQs
1. which one of the following drugs works by inhibition TNF
A. cyclosporine
B. infliximab
C. methotrexate
D. azathioprine
E. sulphasalazine
2. which one of these diseases are less likely to be presented with acute
monoarthritis
A. septic arthritis
B. gouty arthritis
C. pseudo gout
D. rheumatoid arthritis
E. traumatic arthritis

28
3. the most common pulmonary complication in patient with systemic
sclerosis is
A. pulmonary fibrosis
B. pleural thickening
C. pleural effusion
D. bronchogenic carcinoma
E. lung nodules
4. rheumatoid factor present in which of the following conditions
A. ankylosing spondylitis
B. osteoarthritis
C. psoriatic arthritis
D. sjőgrenʼs syndrome
5. which of the following is true regarding giant cell arteritis
A. high C reactive protein
B. visual loss with blindness is a significant cause of morbidity
C. common at age of 20 years
D. peripheral retinal pigmentation are characteristic
E. commonly asymptomatic and discovered on routine imaging
6. which one of the following antibodies is most specific for limited
cutaneous systemic sclerosis
A. anti-jo1 antibodies
B. anti-scl-70 antibodies
C. rheumatoid factor
D. anti-centromere antibodies
E. ANA
7. which one of the following is not seen in reactive arthritis
A. urethritis
B. keratoderma blenorrhagica
C. conjunctivitis
D. aseptic meningitis
E. circinate balanitis

29
8. the treatment of choice in a patient with severe deforming psoriatic
arthritis is
A. Etanrercept (TNF alpha inhibitor)
B. hydroxychloroquine
C. prednisolone
D. sulphasalazine
9. in drug induced lupus which of the following statement is correct
A. disease persistent even when discontinues of drugs
B. anti-ds-DNA positive
C. CNS and kidney are usually involved
D. anti-histones antibodies
E. the female to male ratio are 9:1
10. which one of the following antibodies is associated with mixed
connective tissue disease
A. anti-Ds-DNA
B. anti-RNP
C. anti-centromere antibodies
D. anti-neutrophilic cytoplasmic antibodies ANCA
E. rheumatoid factor
11. all the following are characteristic features of systemic sclerosis
except
A. female predominance
B. pulmonary hypertension
C. Raynaud's phenomenon
D. positive anti-Jo1 antibodies
E. sclerodactly
12. which of the following drugs is most likely to cause SLE
A. baclofen
B. sulfasalazine
C. azathioprine
D. procainamide

30
E. methotrexate
13. which of the following true about gout
A. acute gout tend to be poly-articular
B. may be associated with DM
C. allopurinol treatment of choice in acute gout
D. asymptomatic hyperuricemia need to be treated with allopurinol
14. one of the following is not clinical features of Bechet disease
A. recurrent vein thrombosis
B. posterior uveitis
C. nephrotic syndrome
D. erythema nodosum
E. arterial aneurysm
15. which of the following auto-antibodies is most strongly associated
with anti-phospholipid syndrome
A. ANA
B. anti-parital cell antibody
C rheumatoid factor
D. anti-cardiolipin antibodies
E. anti-histones anti-bodies
16. which of the following statement regarding pseudo gout is true
A. X-ray of joints are commonly normal
B. do not cause polyarthritis
C. the is affected in over 50% of all acute attacks
D. joint fluid examination will show more than 90% of neutrophilic leukocytosis
E. involvement of sacroiliac joints
17. typical feature of seronegative sponylarthritis include all except
A. asymmetrical oligoarthritis
B. axial joint involvement
C. enthesitis of tendinous insertion
D. uveitis
E. subcutaneous nodules

31
18. regarding drugs side effect, the following associations true except
A. methotrexate and nephrotoxicity
B. infliximab and TB
C. corticosteroids and osteoporosis
D. NSAIDs and peptic ulcer disease
19. the following are recognized associations except
A. enthesitis and ankylosing spondylitis
B. polymyositis and malignancies
C. scleroderma and pulmonary hypertension
D. drug induced lupus and nephritis
E. Bechet's disease and DVT
20. in ankylosing spondylitis, the following true except
A. is more common in male
B. back pain worsen with movement
C. is associated with HLA B27
D. is associated with pulmonary fibrosis
21. clinical feature of systemic sclerosis include all of the following
except
A. Raynaud's phenomenon
B. interstitial lung disease
C. diarrhea and malabsorption
D. cranial nerve palsies
22. all of the following are extra-articular manifestation of ankylosing
spondylitis except
A. apical lung fibrosis
B. anterior uveitis
C. acute glomerulonephritis
D. secondary amyloidosis
23. in reactive arthritis ( Reiter's syndrome) which is false
A. arthritis usually develop weak after gonorrhea or diarrhea
B. dactylitis may occur

32
C. circinate balanitis is recognized manifestation
D. RF is commonly found in high titer
E. sacroilitis may occur
24. feature of Bechet disease include the following except
A. genital ulcer
B. iritis
C. sagittal sinus thrombosis
D. alopecia areata
E. pathergy sign is characteristic
25. which of the following is TRUE about serologic testing in SLE
A. positive ANA is specific for lupus
B. positive anti-RO is specific ffor lupus
C. complement level C3& C4 decrease in active lupus
D. the majority of patient with SLE have anti-SM antibody
26. The followings are features of Behcet disease, EXCEPT
A. Oro-genital ulceration
B. Recurrent deep vein thrombosis
C. Blindness
D. severe pulmonary fibrosis
E. Positive pathergy test
27. Which one of the following features is least commonly seen in drug
induced lupus
A. arthralgia
B. glomerulonephritis
C. Malar rash
D. Myalgia
28. the following are important feature of Bechet's disease except
A. uveitis
B. oral ulcer
C. genital ulcer
D. den venous thrombosis

33
E. thrombocytopenia
29. the following drugs are used for management of lupus nephritis
except
A. frusemide
B. leflunomide
C. cyclophosmide
D. prednisolone
E. azathioprine
30. All of the following therapies are indicated for worsening widespread
systemic psoriatic disease EXCEPT:
A.   Alefacept
B.   Cyclosporine
C.   Infliximab
D.   Methotrexate
E.   Prednisone
31. Rheumatoid arthritis is strongly associated with HLA
A. DR3
B. DR4
C. DR8
D. B27
E. B 9
 

34
Rheumatology cases
1. A 25-year old gives birth to a baby with complete heart block who
subsequently require pacemaker insertion
Which of the following antibodies is most likely to be detected in the
maternal serum
A. anti-ds-DNA
B. anti-scl70 antibodies
C. ANA
D. anti-RO antibodies
E. anti-endomysial antibodies
2. A 36-year old woman present with a 3 month history of progressive
symmetrical polyarthritis affecting wrists, knee, and metacarpophalangeal
joints and morning stiffness. Investigation: ESR 50 mm/ Hr, CRP 100 mg/l,
rheumatoid factor strongly positive
Which is the most appropriate treatment

35
A. systemic corticosteroid
B. NSAIDs and disease modifying agents
C. large dose aspirin
D. broad spectrum antibiotics
3. A 50-year old women presents with dry eyes, dry mouth and polyarthlagia.
Investigation: ANA strongly positive, anti RO strongly positive, rheumatoid
factor positive
What is the diagnosis
A. rheumatoid arthritis
B. sjőgrenʼs syndrome
C. SLE
D. scleroderma
4. A 54-year old man presents with painful proximal myopathy and macular
rash over his back. He is a heavy smoker and his sodium 121 mmol/l
What is the likely underlying diagnosis
A. Addison disease
B. polymyositis
C. dermatomyositis
D. hypothyroidism
5 . A 40-year old lady has painful swollen MCP joints on the left hand and a
swollen right wrist. She has a photosensitive facial rash. Urine analysis reveal
significant protein urea. Rheumatoid factor is negative, but ANA and anti-ds-
DNA antibodies are positive
Which of the following is the most likely diagnosis
A. rheumatoid arthritis
B. systemic lupus erythematosus
C. reactive arthritis
D. mixed connective tissue disease
E. sjogran syndrome
6. A 72-year old man presents with an acutely painful right knee , he is a
known diabetic. On examination, temp. 37 with a hot swollen right knee.
Investigations: WBC 12.6 X 10⁹/L. a knee x-ray revealed reduced joint space
and chondrocalcinosis of the articular cartilage. Culture of aspirated fluid
revealed no growth.

36
What is the most likely diagnosis
A. gout
B. pseudo gout
C. psoriatic monoarthropathy
D. septic arthritis
E. rheumatoid arthritis
7. A 54-year old woman presents with joint pain. She gave a history of
Raynaud's phenomenon. GI upset with diarrhea and difficulty swallowing with
some weight loss. She has some telangiectasia evident on her face.
Investigation positive anti-scl-70 antibodies
Which of the following is the most likely diagnosis
A. mixed connective tissue disease
B. CREST syndrome
C. SLE
D. progressive systemic sclerosis
E. rheumatoid arthritis
8. 57 -year old man comes to emergency department because of severe pain
in his right big toe, O/E temperature 38C ,BP 130/90 mmHg, PR 80/min.
examination shows an erythematous, warm, swollen and tender right big toe.
What is the first optimal investigation should be done to establish
diagnosis
A. serum uric acid
B. rheumatoid factor
C. CRP and ESR
D. synovial fluid analysis
9. 40 -year old obese women with serum uric acid level 8 mg/dl, was obtained
for routine checkup. the other investigations are normal. She eats meat
several time per week, and family history positive for gout. All the examination
is unremarkable
The most appropriate treatment is
A. allopurinol
B. steroids
C. colchicine
D. life style adjustment, dietary control and monitor with blood tests
37
10. A 65-year old female patient C/O right sided hemi cranial headache of 3
months duration, associated with attack of decrease visual acuity right eye.
She had jaw claudication on mastication
The most likely diagnosis is
A. takayasua arteritis
B. Wagner's granulomatosis
C. giant cell arteritis
D. poly arteritis nodos
11. A 36 year old woman presents with a 3 month history of progressive
symmetrical polyarthritis affecting wrists, knees, and metacarpophalangeal
joints and morning stiffness. Investigations: ESR 50 mm/hr, CRP 100 mg/l,
rheumatoid factor strongly positive.
What is the most likely diagnosis
A. Rheumatoid arthritis.
B. Systemic lupus erythromatosus
C. Rheumatic fever.
D. Psoriatic arthritis
E. Reactive arthritis
12. A 27 year old man presented with progressive low back pain and stiffness
for a year. Stiffness was in the morning and improved with exercise. On
examination: He had tenderness of both sacroiliac joints on palpation and a
red right eye. A pelvic x-ray reveals bilateral sacroiliitis.
What is the most likely diagnosis
A. Bilateral sacroiliac joint subluxation.
B. Reactive arthritis.
C. Psoriatic arthritis.
D. Ankylosing spondylitis.
E. Rheumatoid arthritis
13. 50 year old woman presents with dry eyes, dry mouth and polyarthralgia.
Investigations: ANA strongly positive (1:1600), anti-Ro/SSA antibodies
strongly positive, rheumatoid factor positive, IgG markedly elevated, IgM and
IgA are normal. kappa/lambda ratio is normal.
What is the most likely diagnosis
A. Hyper viscosity syndrome.

38
B. Myeloma associated vasculitis.
C. Primary Sjogren's Syndrome.
D. Systemic Lupus Erythematosus.
E Gouty arthritis.
14. A 16 year old girl presents with a 3 month history of polyarthralgia and
marked early morning stiffness. History of photosensitivity facial skin rash with
history of recurrent mouth ulcer and hair loss. On examination: malar rash on
the face, and signs of arthritis at wrist, metacarpophalangeal and proximal
interphalangeal joints. Investigations: ESR 50 mm/hr, CRP 8 mg/1 (normal=
8-10), rheumatoid factor negative, ANA strongly positive (1:1600), anti-dsDNA
antibodies positive.
What is the most likely cause
A. Systemic Lupus Erythematosus.
B. Systemic sclerosis.
C. Fibromyalgia.
D. Rheumatoid arthritis.
E. Wegener granulomatosis

INFECTIOUS MCQs
1. which of the following is true regarding HIV infection
A. CD4 cells is the major target of HIV virus
B. opportunistic infection usually appears at CD4 count below 700
C. only patient with CD4 cells below 350 should be treated
D. HIV /HCV coinfection is uncommon
E. antiretroviral treatment decrease the risk of mother to child transmission to
below 50%
2. regarding fever of unknown origin which of the following statement is
true
A. infection is still the first leading cause
B. drug history is not relevant
C. history of travel to tropical area is mandatory for diagnosis of malaria
D. empirical antibiotic is not indicated in patient with fever of unknown origin
3. which of the following is not correct regarding sepsis

39
A. it is a dysregulation of normal response
B. worldwide sepsis is a leading cause of death
C. despite reduction in incidence mortality rate still high
D. an alteration in mental status is part of qSOFA criteria
E. thee hour bundle in management of sepsis consist of blood culture,
Inotropic and antibiotic
4. which of the following is correct regarding kala azar
A. leishmania aethiopica is the main cause
B. transmitted by tick bite
C. it is diagnosed by microscopic examination of bone marrow or lymph node
aspirate for Amastigote
D. treatment is supportive as it is a self-limiting disease
E. streptomycin for 2 weeks is the treatment of choice
5. which of the following common primary source of sepsis
A. Abdomen / pelvis
B. primary bacteremia
C. pulmonary
D. urosepsis
6. which of the following statement true regarding brucellosis
A. Brucella melitensis is the most virulent and causes the most severe and
acute cases of brucellosis
B. hepatosplenomegaly and lymphadenopathy occur in more than 90% of
cases
C. peripheral blood film is highly sensitive for diagnosis
D. monotherapy with ciprofloxacin for 2 weeks is curative in most patient
7. regarding the diagnosis of typhoid fever, which of the following is true
A. widal test is highly sensitive in first week
B. bone marrow aspirate cultures give the best confirmation of typhoid fever
C. stool and urine culture are usually positive during the first week
D. blood culture is best performed on the third week
E. titer of antibody to O antigen > 1:640 is specific and diagnostic of typhoid
fever

40
8. which of the following is the antimicrobial drug of choice in the
treatment of brucellosis
A. doxycycline and streptomycin
B. Amoxicillin and clarithromycin
C. Ciprofloxacin
D. Cephalosporin
E. sodium stibogluconate
9. regarding brucellosis which of the following is true
a. the causative organism is gram positive aerobic coccobacilli
B. cannot be transmitted with direct contact with infected animals
C. hepatosplenomegaly is rare
D. sacroillitis and endocarditis are recognized feature
E. blood culture is strongly positive in chronic brucellosis
10. regarding leishmaniosis which of the following statement is false
A. is transmitted by infected bite of female sand fly
B. the fly inject promastigote into the skin and represent the infected stage
C. most infections are severely symptomatic
D. Leishmanial infantum is the causative agent of visceral leishmanial
11. regarding SARS-Cov-2 one of the following is true
A. it is a DNA virus
B. around > 50% of infected people may need hospitalization
C. a 50-60% alcohol is found to be effective disinfectant for COVID-19 virus
D. Guillian- berry syndrome is one of the recognized post COVID
complications
E. the blood test is primary diagnostic test for the identification of virus
12. regarding the anti-retroviral therapy in HIV infected subjects, which
one of the following is true
A. it does not affect the risk of sexual transmission of HIV virus
B. it is not recommended in pregnant patient
C. HIV associated nephropathy is an indication to start anti-retroviral therapy
D. hepatitis B co-infection is an indication to stop antiretroviral therapy

41
E. it should be started only if CD4 count is less than 50/mm3
13. regarding fever of unknown origin FUO, which of the following is
true
A. infection is the second leading cause
B. about 10% of cases discharge without definitive diagnosis
C. malaria is rare cause
D. drug induced fever is well recognized cause
E. bone marrow aspiration is diagnostic in almost all cases of FUO
14. which is true regarding pneumocystis jiroveci (carinii) pneumonia
A. chest X-ray finding is pathognomonic
B. seen only in AIDs patient
C. carries a high mortality rate
D. prophylaxis is only indicated if patient had previous attacks
E. treated by antifungal
15. which of the following is true in patient with visceral leishmaniosis
A. serologic tests alone are sufficiently sensitive and specific to definitively
confirm or exclude leishmaniosis
B. most infections with visceral leishmaniosis are a symptomatic
C. it tends to run less severe course in HIV infected patients
D. pentavalent antimonial compounds are curative in more the 90% of cases
E. hypergammaglobulinemia is characteristic
16. what does SOFA stand for
A. san Francisco organ failure assessment
B. sepsis related organ failure assessment
C. severe organ failure assessment
D. single organ failure assessment
E. serious organ failure assessment
17. trimethoprim sulfamethoxazole is DOC in treatment of
A. toxoplasmosis of brain
B. CMV
C. bacterial pneumonia

42
D. pneumocystis carinii pneumonia
E. giardiasis
18. regarding malaria all of the following are true except
A. can be transmitted by blood transfusion
B. can present with convulsion
C. can cause ARF
D. metabolic acidosis is one of complication
E. IV chloroquine is DOC in complicated malaria
19. all of the following are true regarding typhoid fever except
A. human is the only reservoir of the infection
B. gentamicin is the treatment of choice
C. causes intestinal hemorrhage
D. blood culture is important diagnostic method
E. 5% of patient become chronic carrier
20. all of the following are true regarding infectious mononucleosis
except
A. amoxicillin therapy induce skin rash
B. severe pharyngeal edema should treated with steroid
C. IV ciprofloxacin is the treatment of the choice
D. splenic rupture may occur
21. about typhoid fever which of the following is false
A. human is only known reservoir
B. cough, sore throat and altered behavior can be presenting feature
C. relative bradycardia is recognized feature
D. blood culture positive in 50% in first week
E. ciprofloxacin is drug of choice
22. opportunistic infection prophylaxis in AIDs patient usually start at/in
A. any CD count
B. CD count between 300-500/ml
C. asymptomatic patient with CD count < 250/ml
D. CD4 level less than 200/ml

43
E. patient with active tuberculosis
23. regarding brucellosis which of the following is true
A. the causative organism is gram positive coccobacilli
B. cannot transmit by direct contact with infected animals
C. may presented with spastic paraplegia
D. the treatment of choice is penicillin
E. blood culture is often positive in chronic cases
24. regarding HIV which is true
A. most infection are caused by HIV2
B. seroconversion 3-6 months after exposure
C. pneumocystis carini is considered as AIDs defined illness
D. HAART is started only when CD4 < 200

Infectious cases
1. A 40-year-old man presented with fever , constipation and abdominal pain .
he gave history of frequent food intake from poor hygiene
On examination : he is sick, temperature 39.5C, pulse 70bpm,he has rose
spots on abdomen and back and splenomegaly. Other examination was
normal
What is the most appropriate treatment
A. erythromycin
B. Ciprofloxacin
C. Gentamicin
D. Metronidazole
2. A 45-year-old man working in Sudan, presented to E/R complaining of high
grade fever, chills and rigor about two weeks back. His family notes that he
became confused, his urine dark.
On examination he is conscious but disoriented, no sign of meningeal irritation

44
The most likely diagnosis is
A. Brucellosis
B. typhoid fever
C. bacterial meningitis
D cerebral malaria
3. an HIV patient has cough, fever and sputum production for 4 day. A chest
X-ray shows a left lower lobe infiltration
CD4 count 500
The most likely microbial pathogen is
A. Cryptococcus's
B. CMV infection
C. pneumocystis jiroveci
D. streptococcus pneumonia
E mycobacterium tuberculosis
4. A40-year-old man presents with fever , headache, vomiting and confusion,
one month after returning from trip to Sudan. He deteriorated rapidly and
became confused within 2 days of presentation. On examination: he has
pallor and jaundice , no skin rash , no neck stiffness or focal signs.
Investigations: HB 6 gram /dl, WBCs 12.000/mm3, platelet 100.000/mm3.
Reticulocyte 15%, serum bilirubin 5 mg/dl, ALT 115u/l, AST 175u/l, he has
negative HCV, HBV and HIV serology
The most likely diagnosis is
A. septic meningitis
B. tuberculosis meningitis
C. falciparum malaria
D. Kala-azar
E. Cryptococcus meningitis
5. A 25-year-old years old man with fever, night sweats, weight loss and
muscle aches for the past one month he has previously well
Examination is notable for splenomegaly he currently works as shepherd
Which of the following is correct regarding this case
A. bone marrow culture has sensitivity of 20%
B. treated by combination therapy of doxycycline 100mg for 2 weeks and
streptomycin 1g/day for 2 weeks
45
C. involvement of central nervous system requires treatment for 2 months
D. endocarditis cases don’t require surgery
E. a minimum of fourfold rise in serum agglutinins is required for diagnosis
6. A 40-year-old man presented with fever, constipation and abdominal pain
for the last two weeks, he gave history of frequent food intake from poor
hygiene restaurant , physical examination : he is sick temperature of 39.5C,
pulse 70/min, he has rose –red spots on abdomen and back and
splenomegaly
Which of the following is the most appropriate treatment
A. erythromycin
B. ciprofloxacin
C. gentamicin
D. metronidazole
E. doxycycline
7. A 18 years female was presented with fever , weakness, vomiting and
yellow discoloration of sclera. Investigations; ALT 1800U/ml, AST 750U/ml,
positive both anti-HAV IgM and anti-HAV IgG. HB-s-Ag and HCV-Ab both are
negative
What is the most likely diagnosis
A. acute hepatitis A(icteric type) and HB-s-Ag
B. acute hepatitis C
C. chronic active hepatitis B
D. acute hepatitis A
E. hepatitis D infection

46
Hematology MCQs
1. regarding chronic lymphocytic leukemia which of the following is true
A. it is a clonal malignancy of T lymphocytes
B. it is a disease affecting mainly young people
C. all patient should be treated with chemotherapy
D. autoimmune hemolytic anemia is a recognized complication
E. the hallmark of the disease is the presence of bcr /abl gene
2. regarding sickle cell disease which of the following is not correct
A. splenomegaly is characteristic
B. painful episodes affecting bone and chest are recognized complications
C. patient may present with stroke
D. hemoglobin S is the major hemoglobin seen in HB electrophoresis
3. which one of the following is diagnostic of Aplastic anemia
A. abnormal blast cell on the blood film

47
B. hypo cellular bone marrow with fat replacement
C. reticulocytosis
D. hypochromic microcytic anemia
E. severe thrombocytopenia
4. regarding non Hodgkin lymphoma which one of the following is
correct
A. almost 85% of non-Hodgkin's lymphoma are derived from T cell
B. more than 80% of non-Hodgkin's lymphoma s occurs in the age of groups
of less than 35 years
C. patient with congenital or acquired immunodeficiency have an excellent
response to therapy
D. lymphoma of large lymphocytes have more indolent course than those of
small lymphocytes
E. the 5 year relative survival rate of patient with NHL is 71%
5. which one of the following recognized feature of myelofibrosis
A. absence of splenomegaly and lymphadenopathy
B. leucoerythroblastic blood picture with tear drop
C. increased leukocyte alkaline phosphatase
D. increased folic acid and hyperuricemia
E. absent megakaryocyte in bone marrow with thrombocytopenia
6. which is the gold standard test to diagnose iron deficiency anemia
A. microcytic hypochromic RBCs
B. low serum iron
C. low serum ferritin
D. decrease total iron binding capacity
E. low reticulocytes
7. which of the following statement is true regarding the presentation of
the patient with acute leukemia
A. gum hypertrophy is seen in patient with acute lymphocytic leukemia
ALL˃AML
B. lymphadenopathy seen in AML˃ALL
C. testicular involvement seen in ALL˃AML

48
D. hepatosplenomegaly seen in AML>ALL
8. prothrombin time is prolonged in all the following except
A. DIC
B. factor VII deficiency
C. factor VIII deficiency
D. warfarin therapy
E. hepatic failure
9. the following contribute to the mechanism of renal failure in multiple
myeloma except
A. monoclonal free light chain
B. nephrocalcinosis
C. hyperuricemia
D. advanced glycation end products
10. all the following is characteristic laboratory result of DIC except
A. decrease platelet count
B. prolonged APTT
C. increase fibrinogen
D. prolonged PT
11. regarding iron deficiency anemia, which of the following is true
A. low serum transferrin
B. is a rare cause of anemia
C. dementia and neuropathy are recognized clinical feature
D. microcytic, hypochromic, anisocytosis and pencil cell in peripheral blood
film
E. high serum ferritin
12. regarding ITP which of the following is false
A. splenectomy is contraindicated
B. it is associated with SLE
C. hyper-cellular bone marrow is characteristic
D. spontaneous bleeding occurs with platelet count fallen below 20.000/ml
E. HBV, HCV. And HIV should be excluded

49
13. which of the following in patient with Hodgkin disease indicate the
worst prognosis
A. 25 year old male with inguinal lymphadenopathy
B. 25 year old female with inguinal and mediastinal lymphadenopathy
C. 25 year old female with inguinal, mediastinal lymphadenopathy and night
sweats
D. 25 year old male with inguinal, mediastinal lymphadenopathy and pruritus
14. all of the following are recognized feature of iron deficiency anemia
except
A. low serum ferritin
B. poikilocytosis
C. low TIBC
D. anisocytosis
E. pencil cells
15. which of the following is not an advantage of low molecular weight
to unfractionated heparin
A. can be given once or twice daily
B. reduced risk of osteoporosis
C. half-life of 30 min, so easily reversed
D. doesn't need monitoring by blood test
E. less thrombocytopenia
16. which of the following is treatment of choice for asymptomatic
patient with low grade non Hodgkin lymphoma
A. radiotherapy
B. chemotherapy
C. radio and chemotherapy
D. only rituximab
E. wait and watch
17. in autoimmune hemolytic anemia the following are true except
A. in warm antibody type, the responsible immunoglobulin is IgG
B. Comb's test is almost always positive
C. peripheral blood smear shows microspherocyte

50
D. autoimmune thrombocytopenia is a recognized association
E. splenectomy is an option in the management
18. HBA-2 more than 4% is found in
A. beta thalassemia major
B. beta thalassemia intermedia
C. beta thalassemia trait
D. alpha thalassemia major
19. in Hodgkin lymphoma which of the following is true
A. the abnormal cell of origin is a clonal T-lymphocyte
B. lymphocyte depleted HL carries the worst prognosis
C. HL is incurable in over 70% of cases
D. stages IIB, III, IV are often treated with radiotherapy
E. disease relapse after many years is only curable by ablative cytotoxic
therapy and bone marrow transplantation
20. which of the following drugs not used in a patient with multiple
myeloma who are fit for bone marrow transplantation
A. melphalan
B. thalidomide
C. bortezomib
D. dexamethasone
E. lenalidomide
21. all the following are features of multiple myeloma except
A. osteolytic lesions
B. increase in lymphoplasmocytoid in bone marrow
C. monoclonal band in serum protein electrophoresis
D. hypercalcemia
E. renal impairment
22. which of the following is true regarding pancreatic carcinoma
A. 10% are adenocarcinoma
B. metastasize to regional LN at late stage
C. localized disease at the time of diagnosis

51
D. 60% Occur at head of pancreas
E. more common in female
23. in a patient with locally advanced esophageal cancer, which is NOT
CORRECT
A. chemotherapy can be used as neoadjuvant chemotherapy or definitive
treatment in squamous cell carcinoma
B. perioperative chemotherapy is an option in adenocarcinoma
C. perioperative chemotherapy should be used only in patient with good
performance status
D. Surgical resection followed by adjuvant chemotherapy is the best treatment
in both squamous and adenocarcinoma
E. c-TI N0 M0 and c-T2 N0 M0 are not considered as locally advanced
disease
24. in a subset of patient with advanced or metastatic disease, which
one of the following has curative potential with primary induction
chemotherapy
A. germ cell cancer
B. non-small cell lung cancer
C. gastric cancer
D. colorectal cancer
E. breast cancer
25. regarding ITP
A. abnormal PLT life span
B. huge splenomegaly
C. poor response to corticosteroid therapy
D. LAP
E. absence of megakaryocyte in bone marrow aspiration
26. The followings are features of anemia of chronic disease except:
A .Mild normocytic normochromic anemia.
B. Occurs in the setting of chronic inflammation and neoplasia.
C. Low serum iron.
D. Low serum ferritin
E. Low serum TIBC "total iron binding capacity"

52
27. The followings are features of hemolysis except:
A. High serum bilirubin.
B. High serum LDH.
C. High reticulocyte count.
D. High haptoglobin
E. High urinary urobilinogen.
28. Which of the followings causes prolonged APTT activated partial
thromboplastin time:
A. Uremia.
B. Unfractionated heparin therapy
C. Aspirin therapy.
D. Idiopathic thrombocytopenic purpura.
E. Essential thrombocythaemia.

Hematology Cases
1. A 33-year- old woman has low grade fever, night sweats, and generalized
malaise for 2 months. On physical examination : she has non tender cervical
lymphadenopathy. Lymph node biopsy shows Reed-Sternberg cell along with
large and small lymphocyte and bands of fibrosis. CD30 positive and CD15
positive lymphocyte
Which of the following the most likely diagnosis
A. non-Hodgkin's Lymphoma
B. Hodgkin's lymphoma
C. HIV infection
D. tuberculous lymphadenitis
2. A man of 22 years has a painful swollen knee joint , investigation a
prolonged partial thromboplastin time PTT, a normal prothrombin time PT
Which is the most likely diagnosis
A. Von willebrand's disease
B. factor VII deficiency

53
C. factor VIII deficiency (hemophilia)
D. vitamin K deficiency
3. A 60-year-old female patient presented with anemia. On examination she
had positive Babinski sign in both sides with absent ankle jerks and absent
vibration sense. Investigation: Hb 6gm/dl, MCV 115fl, MCHC 30, WBC
2500/ml, platelets 80000/ml ,reticulocyte count 0.1%, lactate
dehydrogenase750IU/dl
Which of the following is the most appropriate management
A. oral folate replacement
B. vitamin B12 and folate
C. packed red blood cells transfusion
D. whole blood transfusion
4. All the following are associated with this disease except
A. reversible dementia
B. optic atrophy
C. primary hypothyroidism
D. cerebellar signs
5. A 73-year-old man presented with headache and dizziness .He is smoker,
and recently had deep vein thrombosis right popliteal vein. By examination :
blood pressure 190/100 mmHg other examination was normal . investigations:
Hb 19g/dl, WBC 12000/ml, and platelet 500.000/ml, hematocrit 58(N40-57)
Which of the following is the most appropriate investigation to establish
diagnosis
A. abdominal ultrasound scan
B. arterial blood gas
C. reticulocyte count
D. bone marrow biopsy
E. red blood cell mass
6. A 45-year-old female Libyan patient admitted to the hospital because of
anemia and huge splenomegaly . investigation: HB 7g/dl, WBC 80.000/ml,
and platelet count of 70.000/ml. blood film shows a normocytic normochromic
anemia ,and full range of granulocyte precursor with increase in mature
neutrophil. Cytokines study showed fusion gene bcr/abl
What is the most likely diagnosis
A. chronic myeloid leukemia

54
B. myelofibrosis
C. non-Hodgkin's lymphoma
D. thalassemia
7. A 26-year-old lady presented with generalized fatigability. She reports that
she had history of menorrhagia and easy bruising. Investigation: low serum
iron, microcytic hypochromic anemia, normal platelets, coagulation profile:
normal PT. prolonged APTT. Prolonged bleeding time
Which of the following is the most likely diagnosis
A. antiphospholipid syndrome
B. Anti-thrombin III deficiency
C. hemophilia
D. Von willebrad's disease
8. A 73-years-old hypertensive woman presented with splenomegaly.
Investigations: Hb 11g/dl, WBC 8000/ml, and platelet 970.000/ml
Which of the following is the most likely diagnosis
A. polycythemia rubra Vera
B. Chronic lymphocytic leukemia
C. essential thrombocytosis
D. acute lymphocytic leukemia
9. A 50-year-old female presented with 6 week history of tiredness and
abdominal discomfort. On examination she is pale with huge splenomegaly.
Her hemoglobin 7.2 g/dl, WBC 100.000/ml, plt 60.000/ml. her peripheral blood
film shows neutrophilia with left shift, numerous myelocytes with 4%
myeloblast. Cytogenic study shows reciprocal translocation of chromosome
9:22
Which of the following is the first line treatment for this patient
A. Imantinib 400mg orally
B. allogenic sibling matched donor stem cell transplantation
C. hydroxicarbamide 500mg orally
D. autologous stem cell transplantation
E. subcutaneous interferon 4.5 million units
10. A 35-year-old female presented with pallor and jaundice. She had no past
medical history of same disease, no family history of blood disease. CBC
showed Hb 6g/dl, MCV 95 fl ( N 76-96), WBC 5.2X 109, platelet 200X109.

55
Reticulocyte count 5%. Peripheral blood smear shows sperocytosis and
nucleated RBCs
The best next appropriate lab test to confirm diagnosis
A. serum vitamin b12 level
B. serum folate level
C. Coomb's test
D. bone marrow aspiration
E. USS abdomen
11. 28-years-old lady with free past medical history presented few days with
generalized fatigue, exertional dyspnea, poor appetite, gum bleeding,
epistaxis, bleeding per vagina and ecchymosis over upper and lower limbs.
On examination, ill looking, pale, multiple echymotic patches, bleeding gums.
WBC count 13X103 /ml, Hb 5 g/dl, platelet 20X103 /ml. PBF showed 30%
blast with Auer rod, MPO positive. Microangiopathic hemolytic anemia. Serum
uric acid 5mg/dl and normal LFT, RFT. Serum calcium 8 mg/dl. PT and APTT
both are prolonged and fibrinogen low
Which of the following drugs may be included with induction
chemotherapy
A. allopurinol
B. systemic antibiotics
C. Imatinib
D. All-trans retinoic acid ATRA
E. rituximab
12. A 70-years-old man presents with weakness of his right arm and leg. His
symptoms began one day before presentation and are now resolved. He also
reports a 6-months history of recurrent headaches and fatigue. Findings on
physical examination are normal. CBC revealed Hb 20.5 g/dl, WBC
12.500/microliter, and PLT 600.000/microliter
Which of the following should you order next
A. JAK2 V617F mutation testing
B. ABG analysis
C. bone marrow aspiration and biopsy
D. computed tomography scanning
E. florescence in situ hybridization FISHI for BCR-A4BL testing

56
13. at her annual physical examination, an asymptomatic 68-year-old woman
has lymphocytosis ( 32.000/microliter) with a normal hemoglobin level and
platelet count. On examination she has 1 cm lymphadenopathy in the cervical
region and no palpable liver or splenic enlargement. Further work up is
consisting with chronic lymphocytic leukemia CLL
Which of the following is the best step in her management
A. combined chemo immunotherapy
B. chlorambucil therapy
C. allogenic bone marrow transplant
D. active monitoring for disease progression and complications
E. cyclophosphamide and rituximab should be started
14. A 55-year old female presented with intermittent constipation and diarrhea
associated with abdominal cramping. Colonoscopy showed near obstructing
mass in sigmoid colon . she underwent exploratory laparotomy and resection
of 5X5cm fumigating mass. Pathology revealed adenocarcinoma invading into
pericolonic fat with 2 of 30 positive lymph nodes. CT scan showed no
evidence of metastasis
After recovery from her surgery, which of the following would you
recommended
A. adjuvant chemotherapy with5-FU and oxaliplatin
B. chemo radiotherapy
C. radiotherapy
D. hormonal therapy
E. follow up every 6 months
15. 22-year-old male student admitted with weakness and tiredness for last 3
weeks. He has been otherwise well. On examination, petechial rash on lower
legs and conjunctival pallor. He takes no medications and denies any illicit
drug use. Investigations revealed: HB 4g/dl, WBC 1x109/l, PLT 20X 109/l,
normal clotting profile and normal LFT, RFT
The most likely diagnosis is
A. acute myeloid leukemia
B. acute lymphocytic leukemia
C. aplastic anemia
D. Hodgkin lymphoma
16. 70-year-old man presented for medical checkup he is found to have
generalized LAP , but no pallor, jaundice or hepatosplenomegaly. Lab shows

57
Hb 13 g/dl, WBC 25.000/ml with 90% lymphocyte & 10% neutrophil, PLT
225.000/ml, PBF mature appearing lymphocyte with smear cells.
What is the most likely diagnosis
A. ALL
B. NHL
C. CLL
D. multiple myeloma
17. A 40-year-old man presented to his GP with fatigue and also complains of
numbness and tingling in the lower limbs. He recalls having an operation on
his abdomen a few years ago while abroad for severe abdominal pain and
upper GI hemorrhage, but cannot recall the specifics of the surgery.
Otherwise, he has been fit and well. Examination reveals pale conjunctivae
and a lemon-yellow tinge to his skin. He has slight distal lower limb sensory
loss and absence of ankle jerks with exaggerated knee reflexes. Abdominal
examination shows a longitudinal median scar. but no organomegaly.
Investigations; Hb 10.4 g/dI, WBC 4,5 x 103/dl , PLT 250 x 10 3/
dl, MCV112 fl (N 76-96f1)
Which operation is the most likely to have had
A. Hemicolectomy.
B. Terminal ileac resection.
C. Pancreaticoduodenectomy.
D. Splenectomy.
E. Partial gastrectomy
18. A 65-year-old lady attends the Emergency Department complaining of
increasing lower back pain, after a weekend of gardening. Her symptoms are
relieved with ibuprofen. She denies any bowel or bladder symptoms, and is
able to weight bear, On examination, heart sounds are normal, the chest is
clear, no organomegaly. Cranial and peripheral nervous system examination
are normal. Investigations: Normocytic normochromic anemia, WBC and
Platelets were normal, mild renal impairment, Albumin 32 g/l (37-49), Alkaline
phosphatase 55 U/l (45-105), AST 20 U/ (1-31), total protein 85 g/1 (61-76),
corrected Calcium 2.7 mmol/l (2.2-2.6), urine protein ++
Which of the following is the most likely diagnosis
A. Multiple myeloma.
B. Granulomatosis with polyangiitis.
C. Polyarteritis nodosa.

58
D. Anti-glomerular basement membrane (GBM) disease.
E. Eosinophilic interstitial nephritits.

GIT MCQs
1. Barrett's esophagus is defined as
A. A condition in which squamous epithelial cells changed to columnar cells
B. A condition in which columnar epithelium changed to squamous epithelium
C. intermittent dysphagia is one of presenting symptoms
D. histological examination is usually not indicated
E. surgical treatment indicated in high grade dysplasia
2. which of the following feature is suggestive of Crohn's disease
A. less common in smokers
B. skip lesions is characteristic of the disease
C. inflammation is limited to mucosal layer
D. extra intestinal manifestation is uncommon
E. fistula are rarely seen

59
3. which of the following test is the best marker of liver function
A. serum alanine transaminase
B. serum alkaline phosphatase
C. platelet count
D. prothrombin time
E. platelet count
4. which of the following is typically raised in patient with primary biliary
cirrhosis
A. anti-mitochondrial antibody
B. anti-smooth muscle antibody
C. serum LDH
D. serum calcium
5. which of the following is suggestive of ulcerative colitis than in
Crohn's disease
A. presence of iron deficiency anemia
B. terminal ileum ulcers
C. elevated P ANCA
D. elevated ASCA
6. regarding malabsorption syndrome which one of the following is true
A. serum anti-gliadins and anti-endomysia antibodies are diagnostic of
Crohn's disease
B. iron deficiency anemia is one of the manifestation of adult celiac disease
C. the presence of peripheral neuropathy exclude the diagnosis of celiac
disease
D. schilling test is used to test vitamin B6 deficiency
E. serum triglycerides and cholesterol are usually elevated
7. A malignant cause of dysphagia is most specifically associated with
which one of the following
A. history of chronic reflux esophagus
B. history of barrettes metaplasia
C. history of regurgitation
D. dysphagia predominately for liquids

60
E. history of melena
8. hepatic encephalopathy precipitated by all the following except
A. SBP
B. hypokalemia
C. hyperglycemia
D. constipation
E. upper GIT bleeding
9. the following are true regarding IBS except
A. abdominal pain which is relieved with defecation
B. episodes of diarrhea alternating with constipation
C. excessive passage of mucous and blood per rectum
D. no weight loss
10. Crohn's disease is associated with all the following except
A. cholelithiasis
B. pyoderma gangrenousm
C. erythema marginatum
D. urinary oxalate stones
11. which of the following causes diarrhea and clubbing
A. intestinal tuberculosis
B. small intestinal malignancy
C. celiac disease
D. diabetic gastroparesis
12. according to Rome III criteria for diagnosing irritable bowel
syndrome which of the following is true
A. diarrhea and weight loss over the last one month
B. abdominal pain and diarrhea occur once monthly over the last three
months
C. recurrent abdominal pain and bloody diarrhea over one month
D. recurrent abdominal pain and diarrhea that is relieved by defecation almost
daily over the last three months
E. abdominal pain and bloody diarrhea over the last three days

61
13. which of the following precipitate hepatic encephalopathy
A. lactulose
B. rifaximin
C. spontaneous bacterial peritonitis
D. esophageal varices ligation
14. which of the following is true regarding Crohn's disease
A. less common in smoker
B. sparing of small intestine
C. in adult age group male commonly affected more than female
D. presence of vesico-colonic fistula
E. it is a common disease in rural than urban areas
15. in acute diarrhea which of the following is indicative of colonic
infection
A. post prandial diarrhea
B. normal temperature
C. large volume diarrhea
D. blood in stool
E. elevated fecal fat
16. alcoholic hepatitis is associated with
A. leukopenia
B. typical acneiform rash
C. predominantly raised AST
D. disease progression despite alcohol cessation
E. markedly elevated ALT
17. the following are high risk for developing colorectal carcinoma
except
A. long standing inflammatory bowel disease
B. diverticulosis
C. prior history of colon cancer
D. presence of large adenomatous polyp
E. familial adenomatous polyposis

62
18. in primary biliary cirrhosis which of the following is recognized
feature
A. lymphadenopathy
B. hemolytic anemia
C. vitamin D,A,K deficiency
D. vitamin B12 deficiency
E. constipation
19. in gastro esophageal reflux disease which of the following is false
A. dysphagia is always indicative of benign stricture
B. symptoms may improves with smaller meals and maintaining upright
position postprandial
C. upper GI endoscopy may reveal reflux esophagitis
D. proton pump inhibitors are usually effective in controlling symptoms
E. Barrett's metaplasia may be found in some patient
20. regarding dysphagia all of the following are true except
A. dysphagia at level of lower sternum suggest lower esophageal disease
B. commonly occurring in bulbar and pseudobulbar palsy
C. painless dysphasia exclude esophageal tumors
D. Achalasia could be the cause
E. dysphagia to fluid suggest neurological disorder
21. regarding Celiac disease the following are true except
A. it never occur in adult
B. serum anti-endomysium IgA antibody titer are characteristically elevated
C. gluten free diet improve absorption and villous atrophy
D. there is predisposition to gut lymphoma and carcinoma
E. the toxic agent is polypeptide alpha gliadin
22. all are parameters of CHILD-PUGH grading for liver cirrhosis except
A. ascites
B. serum albumin
C. serum bilirubin
D. prothrombin time

63
E. liver transaminases
23 . regarding management of ulcerative colitis all of the following are
true except
A. screening for colorectal cancer is recommended
B. distal disease can be treated with mesalasine enema
C. steroid are recommended for long term use
D. oral mesalasine is effective for maintaining remission
E. treatment with infliximab may reactive tuberculosis infection
24. all of the following concerning HBV infection are true except
A. the majority of infections are self-limited
B. the persistence of HBsAg for long time indicate chronicity
C. most patient are discovered by routine screening
D. it can be prevented by vaccination
E. it does not lead to chronic liver disease unless associated by previous HDV
infection
25. regarding the esophageal reflux the following is true
A. chronic cough and asthma is one of the main symptom
B. ambulatory pH monitoring is indicated in all patient
C. endoscopic examination normal in all patient
D. characterized by increase gastric emptying
E. surgical treatment is indicated in extra esophageal manifestation
26.regarding helicobacter pylori eradication therapy in peptic ulcer
disease which one of the following is true
A. successful eradication significantly reduce peptic ulcer disease recurrence
rate
B. eradication therapy is indicated in duodenal ulcer as a second line therapy
after failure of proton pump inhibitors
C. after successful eradication lifelong acid suppression therapy is
recommended
D. metronidazole based regimens carry an excellent eradication rate in
developing countries
E. dual regimen with omeprazole and clarithromycin is effective in H-pylori
eradication

64
27. SBP all are true except
A. neomycin is the drug of choice
B. is usually caused by gram negative bacteria
C. it should be suspected in any patient with ascites and evidence of clinical
deterioration
D. a raised poly morph nucleus count alone is sufficient evidence to start
treatment immediately
E. it return in > half of patient within one year
28. Wilson disease associated with the following except
A. hemolytic anemia
B. RTA ( renal tubular acidosis(
C. DM
D. blue nail
E. abnormal movement
29. in viral hepatitis which is false
A. hepatitis E high risk of mortality in pregnancy
B. increase risk of HBV &HCV infection in hemophilia patent
C. 50% of adult infected with HBV became chronic carrier
D. there's increase risk of hepatocellular carcinoma in HBV
E. HAV transmitted by feco-oral route
30. regarding HBV infection which is false
A. presence of HB-e-Ag indicate infectivity
B. INF-alpha used in treatment
C. presence of Anti HB-s indicate immunity
D. co-infection with HDV is usually mild
E. the majority of infection is self- limited
31. colorectal cancer all of the following are true except
A. most common GIT cancer
B. commonly arise from benign adenoma
C. incidence decrease by good screening test
D. usually detected by carful abdominal palpation

65
E. has familial tendency
32. Irritable bowel syndrome is associated with all of the following
except:
A. Weight loss.
B. Recurrent colicky abdominal pain.
C. Feeling of incomplete defecation.
D. Rectal mucous.
E. Stress worsens symptoms.
33. Which of the following features is more common in Crohn's disease
than in ulcerative disease
A. Frequent bloody diarrhea.
B. Uveitis.
C. Perianal abscess.
D. Inflammation limited to mucosa.
E. Massive hemorrhage.
34. Which of the following laboratory results is not correct for acute
Hepatitis- B infection
A. Positive HB-S-Ag
B. Positive anti HBC IgG
C. Positive ant i HBC Ig M
D. Negative anti HBs
E. Positive HB-e-Ag

66
GIT Cases
1. A 65 year old man presented with tiredness for many years and know
diagnosed with anemia, he has no past medical history and take no
medications. No weight loss, no change in bowel habit. He had varied diet
including red meat. CBC revealed microcytic hypochromic anemia and low
serum ferritin RFT ,LFT were normal
Which of the following investigations would you request for him
A. barium enema
B. stool for occult blood
C. erect abdominal x ray
D. serum B12
2. A 28 year old female teacher presented with intermittent bloating,
abdominal pain and weight loss. She tells that she had diarrhea that is floating
on the toilet pan and difficult to flush.
Investigation low serum albumin and microcytic hypochromic anemia

67
Which one of the following investigation is useful to perform next for
diagnosis
A. helicobacter pylori serology
B. serum B12 level
C. tissue transglutaminase antibody
D. gastrin level
3 . A 35-year-old female presented with history of yellowish discoloration of
sclera , abdominal pain and weight loss for 8 weeks. Her past history was
positive for infertility. The examination shows jaundice , scratch mark and
hepatomegaly. Blood test were positive for ANA and anti-smooth muscle
antibody
Which of the following is most appropriate next step in management
A. lever biopsy and immunoglobulin level followed by oral prednisolone
B. liver biopsy, prednisolone and azathioprine
C. methyl prednisolone followed by oral prednisolone therapy
D. injectable prednisolone followed by cyclosporine
E. ultrasound and liver biopsy
4 . A 26-year-old man has a one week history of increasingly severe bloody
diarrhea, tenesmus, abdominal pain and fever. On examination he is ill, pale
and dehydrated. Abdominal examination: abdomen is distended with
tenderness. Investigations: hemoglobin 5g/dl, WBC 13.000/ml, ESR 60
mm/hr. plain x-ray abdomen film reveal transverse colon dilatation of 7 cm
Which one of the following is correct regarding treatment of this
condition
A. intravenous cloxacillin is the antibiotic of choice
B. it is recommended to take at least 4 meals and 2 liters fluid orally to
prevent dehydration
C. urgent surgical intervention is indicated if patient fails to improve within 48-
72 hours
D. antibiotics should be given according to stool culture results only
E. urgent colonoscopy is recommended during the first 24 hour
5. A 30-years-old man complain of diarrhea and weight loss. Sigmoidoscopy
showed inflamed friable mucosa. a rectal biopsy is taken and the histology
shows mucosal ulcers with inflammatory infiltrate, crypt abscess with goblet
cell depletion
Which is the most likely diagnosis describing the histology report

68
A. Crohn's disease
B irritable bowel syndrome
C. pseudomembranous colitis
D. ulcerative colitis
E. non-specific colitis
6. A 70-year-old man was admitted with pallor , loose dark stool. Examination
revealed a pulse of 110 BPM, BP 90/50 mmHg. Investigation revealed: Hb
7.2g/dl N( 14-16), MCV 72fl (N 78-96), WBC 11.3 (N 4-11) platelets 480 (N
150-400)
What is the most appropriate next step in his management
A. barium meal
B. blood transfusion
C. endoscopy
D. parenteral iron transfusion
E. proton pump inhibitor therapy
7.A 40-year-old female patient presented with sudden severe epigastric pain
radiating to the back, associated with nausea and vomiting, no hematemesis.
Past history : multiple gall bladder stones. By examination: she is in pain,
general examination is normal, abdominal examination: mild epigastric
tenderness, no organomegaly, bowel sound normal
What important first investigation should be ordered
A. ultra sound scan abdomen
B. serum amylase and lipase
C. CT scan abdomen
D. ERCP
E. upper GIT endoscopy
8.22- year-old female presented with tremors. Family revealed that her aunt
died because of liver cirrhosis. Examination revealed jaundice and tremor.
Investigations: AST 1600U/L (N 1-31), ALT 900U/L(N 5-53), bilirubin 8mg/dl
(N0.4-1.2), albumin 2.8g/dl (N 3.5-4), WBC normal, Hb 8g/dl, reticulocyte
count 5%, platelet normal, PT 50s(N 11.5-15.5), serum cupper 10
micromole/L(N 0-1.6)
What is most likely diagnosis
A. hemochromatosis
B. portal vein thrombosis

69
C. Wilson disease
D. acute fatty liver
E. fulminant hepatitis A
9. A 52-year-old female presented with confusion of few hours duration. She
had developed progressing abdominal distension over the last few weeks.
Examination was remarkable for pallor, ecchymosis, palmar erythema,
bilateral leg edema and dull abdominal percussion note. She had flapping
tremor and she was disoriented to time and person. AST 12(N <35) ALT
16(N<35), Hb 10g/dl, WBC 300/ml, platelet 90.000/ml, INR 2.2, APTT
50s( control 35 second). Abdominal paracentesis revealed PMN leukocyte of
300/mm3
Which of the following is the most appropriate management
A. sodium restriction plus spironolactone 200mg daily plus furosemide
B. repeated therapeutic paracentesis
C. cefotaxime IV
D. IV terlipressin
E. low protein diet
10. A 62-yyear-old female is seen in emergency department after 3 episode of
melena. Her only medication is NSAIDs for long lasting osteoarthritis. In the
supine position, her PR 85/min and her BP 125/80mmHg. On sitting, her PR
115/min and her BP 90/60mmHg.
The first necessary intervention in this patient is
A. upper GI endoscopy
B. vigorous volume replacement
C. gastroenterology consultation
D. discontinuation of NSAIDs
E. IV proton pump inhibitor
11 . A 32-year-old female is evaluated for raised liver enzymes and
generalized fatigue. She had history of jaundice one year earlier and was not
investigated and history of Hashimoto thyroiditis when she was 26 years old.
She was amenorrhic for the preceding 5 months with undetectable serum
beta HCG. Her younger sister diagnosed with celiac disease since childhood.
She takes no medications. Lab results: total serum bilirubin 2.9mg/dl, direct
serum bilirubin 1.9mg/dl, AST 235 and ALT 298 (both normal <35) INR1.1
Which of the following test is helpful in diagnosis
A. serum ceruloplasmin

70
B. serum iron and TIBC
C. serum anti-mitochondrial antibody
D. serum anti-smooth muscle antibody
E. serum alpha-1 antitrypsin
12. A 30-year-old female has a diagnosis of diarrhea predominant IBS. Her
symptoms are negatively impacting her ability to work in a local supermarket
and she is avoiding social functions from fear of unpredictable onset of
symptoms
Which of the following medication is indicated for diarrhea
A. psyllium
B. lubiprostone
C. hyoscynamine
D. loperamide
E. peppermint oil
13 .A 25-year-old female presented with weight loss, abdominal pain and
diarrhea. Her anti-endomysial antibody was positive and duodenal biopsy
confirmed subtotal villous atrophy
Which of the following is a feature of her disease
A. erythema nodosum
B. splenic atrophy
C. colonic ulceration
D. gastric atrophy
14. A 33 year-old male has never been vaccinated for hepatitis B. serological
test reveal HB-surface antigen NEGATIVE and antibody to surface antigen
POSITIVE
Which of the following conditions does this serological pattern best fits
with
A. acute hepatitis B infection
B. chronic active hepatitis
C. previous hepatitis B infection
D. need for vaccination to hepatitis B
15. A 45-year-old female presented with dysphagia to solids and liquids of
gradual onset and progressive worsening over 6 years, associated with
episodes of regurgitation of undigested food, cough and loss of 7 kg despite

71
normal appetite. No history of nasal regurgitation and no history of heartburn,
chest pain or neurological symptoms
The most likely diagnosis is
A. esophageal carcinoma
B. bulbar palsy
C. sliding hiatus hernia
D. achalasia
16. A 40 year old woman presented with vomiting large amount of fresh blood,
4 hours before presentation. On examination: She was markedly pale. Blood
pressure: 80/60 mmHg, pulse: 120/minute .She has flapping tremor, and
ecchymotic skin patches on the lower limbs. The patient was mildly drowsy
but there were no focal neurological signs. Investigations: Hemoglobin: 8
g/dL, Platelets: 105,000/mL - International Normalized Ration INR: 2.2, Renal
function test was normal
What is the immediate management of this patient
A. Slow infusion of hypotonic solutions, consider blood transfusion after 24
hours.
B. Rapid boluses of isotonic crystalloid and rapid infusion of blood substitutes
C. Blood transfusion and fluids should be delayed until upper GIT endoscopy
is done.
D. Slow infusion of isotonic solutions and consider blood transfusion after 24
hours.
E. Refer to surgery for immediate intervention.
F. . Start intravenous iron infusion.
17. A 20-year-old white female presents with malaise and fatigue for 6
months. There is no history of medications, alcohol or drug abuse. There is no
history of previous hepatitis or family history of liver disease. Her physical
examination revealed hepato-splenomegaly and mild jaundice. Investigations:
She had anemia, mild thrombocytopenia. ALT and AST are10 times above
upper limit of normal. Serum albumin low, serum globulins high. Viral hepatitis
A,B, and C serology were negative Anti-nuclear antibodies: Positive and Anti-
mitochondrial antibodies: Negative
Which of the following is most appropriate
A. Begin interferon therapy.
B. Begin daily prednisolone.
C. No further workup or therapy is necessary at this time.

72
D. Begin cyclosporine.
E. Reassurance and follow up in six months.
18. A 30-year old woman had chronic diarrhea and abdominal bloating.
Diarrhea was offensive, and stool floats and is not flushed easily. On
examination: She is emaciated and pale. Investigations: Stool examination
was positive for fat globules, and negative for parasites. Hemoglobin was 8
g/dL, and serum calcium, and albumin were low.
Which one of the following is correct regarding this patient
A. Duodenal biopsy is indicated.
B. Lower Gl endoscopy confirms the diagnosis.
C. This patient's symptoms are likely to be a consequence of large bowel
disease.
D. Presence of mucous and tenesmus is characteristic of this condition.
E. Presence of hematochezia confirms the diagnosis.
19. A 50 year old female patient was admitted to hospital with hematemesis
and melena for one day. She has same illness before. She had pallor,
jaundice; spider nevi on the upper body and multiple scratch markings and
splenomegaly. Investigations showed anemia, thrombocytopenia and mildly
elevated transaminases. Alkaline phosphatase: 450 U/L (normal:<180 U/L).
Viral serology was negative. Abdominal ultrasound showed a normal sized
cirrhotic liver.
What is the most likely diagnosis
A. Chronic hepatitis B infection.
B. Haemochromatosis.
C. Primary biliary cirrhosis.
D. Wilson's disease.
E. Autoimmune hepatitis.
20. A 40 y old female patient presented with sudden sever epigastric pain
radiating to the back, associated with nausea and vomiting, no hematemesis.
Past history: multiple gall bladder stones. By examination: she is in pain,
general examination is normal. Abdominal examination: mild epigastric
tenderness, no organomegaly, bowel sounds normal. Serum lipase was
800 U/L (normal value 10- 140)
What is the most likely diagnosis
A. Acute pancreatitis.
B. Duodenal ulcer.

73
C. Cholecystitis.
D.Gastritis.
E. Gastric carcinoma.

CNS MCQs
1. which one of the following is a non-modifiable risk factor for stroke
A. hypertension
B. diabetes
C. smoking
D. hyperlipidemia
E. race
2. which of the following is recognized feature of posterior circulation
stroke
A. aphasia
B. memory disturbance
C. loss of social recognition
D. ataxia
3. in pseudobulbar palsy which of the following is FALSE
A. exaggerated jaw reflex
B. it is lower motor neuron lesion
C. spastic dysarthria
D. the lesion is usually in the internal capsule bilateral

74
4. which one of the following is not associated with a features of motor
neuron disease
A. respiratory failure
B. fasciculation
C. sphincter dysfunction
D. dysphagia
E. speech disorder
5. which one of the following features is suggestive of Parkinson's
disease
A. poor response to levodopa therapy
B. wide based gait
C. intension tremor
D. bradykinesia
6. idiopathic intracranial hypertension which one of the following is true
A. usually present with hemiplegia
B. diplopia when manifested is usually a false localizing sign
C. papilledema is not typically seen
D. is associated with enlarged cerebral ventricles on CT scan
E. steroid therapy is not a possible etiological factor
7. regarding myasthenia gravis which one of the following statement is
true
A. is caused by antibodies directed against acetylcholine
B may be treated with long term edrophonium therapy
C. wasting of muscle is an early feature
D. may be worsened by aminoglycoside antibiotics
E. thymectomy is always indicated
8. which one of the following is correctly matched
A. hemi neglect and temporal lobe lesion
B. Wernicke's aphasia and fluent speech
C. parkinsonism and action tremor
D. Gillian barre syndrome and hyperreflexia

75
E. cerebellar dysfunction and shuffling gait
9. which one of the following is the prophylactic therapy of choice for
the relapsing remitting form of multiple sclerosis
A. interferon alpha
B. interferon beta
C. interferon gamma
D. tumor necrosis factor
10. which of the following is not a feature of simple partial seizure
A. motor disturbance may involve any body part
B. tingling, numbness are common feature
C. consciousness is impaired
D. flashing light and color usually precede the attack
11. causes of cerebellar ataxia include all of the following except
A. phenytoin
B. chronic alcoholism
C. multiple sclerosis
D. bronchial carcinoma
E. vitamin B12 deficiency
12. in Horner's syndrome all of the following are expected features
except
A. ptosis
B. enophthalmos
C. loss of facial sweating
D. lateral gaze palsy
13. all the following are clinical feature diabetic third nerve palsy except
A. ptosis
B. dilated pupil
C. divergent squint
D. diplopia
14. which of the following neurological signs is most helpful in
differentiating subacute combined degeneration of spinal cord from
multiple sclerosis

76
A. bilateral Babinski sign
B. absent ankle jerk
C. optic atrophy
D. ataxia
E. barber chair sign
15. which of the following is true regarding motor neuron disease
A. typically there is eye involvement
B. peripheral sensory nerve conduction velocities are normal
C. sensory symptoms at onset exclude diagnosis
D. the onset of bulbar palsy indicate good prognosis
E. The function of glossopharyngeal nerve is frequently affected
16. In Migraine which is False
A. Is often relieved by pregnancy
B. Is exacerbated by OCP.
C. Third nerve palsy may seen .
D. Prodromal visual symptoms tend to occur in classical type
E. b and c are True
17. Flapping tremor not seen in :
A. Renal failure .
B. Co2 narcosis.
C. Respiratory failure
D. Parkinsonism.
E. C and A
18. Treatable causes of dementia include all of the following except:
A. Tertiary syphilis.
B. Depression.
C. Alzheimer's disease.
D. Normal pressure hydrocephalus.
E. Vitamin B12 deficiency.

77
CNS Cases
1. A 40-year- old women presented to emergency department with headache ,
blurry vision. Headache was worse over the last two weeks and more sever at
morning, by examination her pulse 40 BPM, BP 190/110, fundus examination
shows bilateral swollen optic disc. she had diplopia on lateral gaze in both
sides
Which of the following is the most appropriate immediate action
A. start thrombolytic therapy
B. perform CT scan of the brain
C. perform lumber puncture
D. start high dose dexamethasone
E. start antiviral therapy
2. A 35-year-old man presented with left hand weakness and a similar 3
months weakness in right hand. On examination he had fasciculation of both
deltoid muscle , he had hypertonia and hyperreflexia both upper limbs,
coordination was normal
What is the most likely diagnosis
A. multiple sclerosis
B. chronic demyelinating inflammatory polyneuropathy

78
C. motor neuron disease
D. vitamin B12 deficiency
3. you asked to see a 77-year- old man with convergent squint and left side
facial weakness
Where is the site of lesion
A. medulla
B cerebellum
C. pons
D. upper spinal cord
4. A 35-year-old woman, presents with symptoms of double vision and
drooping of both eyelids , several weeks ago, more recently she also noticed
heaviness of her legs and arms when climbing upstairs . her symptoms seem
to get worse as the days goes. On examination bilateral partial ptosis, and
proximal muscle weakness
What is the most likely diagnosis
A. myasthenia gravis
B. botulism
C. myositis
D. motor neuron disease
5 A 42-year-old lady had both lower limb weakness over three days
associated with urinary frequency. She had history of dizziness and
unsteadiness over the last 2 weeks. On examination she has slurred speech
and incoordination of the right hand and a spastic Para paresis
Which of the following is the most appropriate investigation
A. MRI brain and spinal cord
B. nerve conduction studies
C. CT scan brain
D. electroencephalogram
6. A 25-year-old lady presents to the emergency department with one hour
history of severe throbbing headache of right supraorbital area . she also
complains of nausea and photophobia. She has had similar attacks in the past
, often brought on by menstruation . about 45 minutes ago she took 400mg of
ibuprofen
Which of the following would be the best therapeutic choice at this time
A. propranolol

79
B. verapamil
C. sumatriptan
D. reassurance no treatment needed
7. A 70-year-old man has progressive difficulty in walking with frequent falls.
On examination he has poor facial expression and hypophonia. He has
increased tone throughout the range of movement of his left upper and lower
limbs with cog wheeling tremor of the left wrist. His CT scan normal
Which one of the following medications should be started
A. risperidone
B. riluzole
C. L dopa & carbidopa
D. atorvastatin
E. pyridostigmine
8. A 17 year-old-male complain of horizontal double vision when looking to the
Left. When he looks to the left and left eye is covered the most peripheral
image disappears
The cause of his diplopia is paralysis of which cranial nerve
A. right sixth nerve palsy
B. left sixth nerve palsy
C. right third nerve palsy
D. left third nerve palsy
E. right fourth nerve palsy
9. A 67-year-old women with history of type II DM and atrial fibrillation
presented to the emergency department with right sided body weakness and
slurred speech of sudden onset. No sensory symptoms, no headache. She is
on warfarin tablet. Physical examination include blood pressure 205/90 and
irregularly irregular pulse rate. There is left side neglect with slurred speech.
There is UMN weakness of right body with face and upper extremity worse
than lower extremity. Routine investigation were normal, INR 1.8
Which one of the following should be done next
A. administer tissue plasminogen activator
B. order a brain CT scan
C. high blood pressure should be vigorously lowered
D. order a cerebral angiogram

80
E. start intravenous heparin
10. A 20-year-old male college student is evaluated for recurrent episodes of
transient loss of consciousness with jerky movement of limbs., he has no
history of head trauma, meningitis, or prior seizure and no family history of
epilepsy. He takes no medication. Results of physical examination, including
neurological examination are normal
Results of laboratory studies, including CBC, serum electrolyte and toxicology
screen, are also normal. An MRI of the brain and EEG show no abnormalities
Which of the following would help exclude the diagnosis of epileptic
seizure in this patient
A. A brief period of tonic-clonic movement a time of falling
B. an aura of strange odor before falling
C. sudden return to normal mental function up on a wakening, though with
feeling of physical weakness
D. cyanotic complexion
E. laceration of the tongue
11. A 17-year-old Libyan girl was admitted with difficulty walking for 2 days
prior to that she had tinging and paresthesia over her limbs. She had history
of URTI one week back. On examination she had weakness of and the upper
limbs and the lower limbs. With loss of all deep tendon reflexes. The planter
response was flexor bilaterally. The weakness of the limbs was more obvious
proximally
Which one of the following is the treatment of choice
A. IV methylprednisolone
B. IV antibiotics
C. pyridostigmine
D. beta interferon
E. IV immunoglobulins
12. A 42 year-old women referred for evaluation of ataxia. Gadolinium
enhanced MRI of brain reveals multiple subcortical white matter lesions as
well as enhancing lesions in the cerebellum and spinal cord. Visual evoked
potentials showed delayed latency op P 100. Vasculitis screen is negative.
Which of the following factors is associated with milder course of this
disease
A. age of 42 year
B. here initial presentation of ataxia

81
C. female gender
D. the MRI scan appearance
13. A 52 year-old-man presented with headache, nausea, vomiting and fever.
On examination is confused, febrile temperature 39C and neck stiffness
Which is the most appropriate clinical approach for this patient
A. lumbar puncture followed by IV antibiotic
B. CT scan then LP if CT scan normal, followed by IV antibiotic
C. IV antibiotic and urgent CT scan head then LP if CT scan normal
D. IV antibiotic till MRI brain performed
E. urgent MRI then antibiotic
14. A 67 year old man who presented having been noted by his family as
developing memory problems for 2-3 years with deteriorating mobility, and
there is number of episodes of wetting himself and being unsure of how this
happened
The most likely diagnosis is
A. Parkinson disease
B. Alzheimer disease
C. normal pressure hydrocephalus
D. motor neuron disease
E. multiple sclerosis
15. 50 year-old alcoholic man is brought to emergency department,
complaining of unsteadiness and confusion. On examination there is
ophthalmoplegia , nystagmus and ataxia
Which of the following medication is the appropriate treatment
A. niacin
B. naloxone
C. thiamine
D. pyridoxine
16. A 32-year-old woman teacher admitted with a three- months history of
transient episodes of double vision. The double vision was worse on looking
up and to the right, The attacks lasted for several hours and tended to happen
more toward the end of the day. On examination there was bilateral partial
ptosis more on the right side, ocular movements were limited in elevation.
Which one of the following tests is most helpful in the diagnosis

82
A. Anti-voltage gated calcium channel antibody.
B. Antinuclear antibody.
C. Anti-acetylcholine receptor antibody.
D. Anti- RO antibodies.
E. Anti -double strand DNA antibodies.
17. A 17 year old Libyan girl was admitted with difficulty walking for 2 days
prior to that she had had tingling and paresthesia over her limbs. She had H/O
upper respiratory tract infection one weak back. On examination she had
weakness of the upper limbs and the lower limbs, with loss of all deep tendon
reflexes. The planter response was flexor bilaterally. The weakness of the
limbs was more obvious proximally.
The most likely diagnosis is which one of the following
A. Syringomyelia.
B. Acute poliomyelitis.
C. Toxic Neuropathy.
D Guillain-Barre syndrome.
E. Myasthenia Gravis.
18. A 32 year old lady had both lower limb weakness over three days
associated with urinary frequency. On examination: she has slurred speech
and incoordination of the right hand and a spastic Paraparesis.
Which of the following is the most appropriates pharmacological
treatment for this patient at this time
A. Beta interferon.
B. IV cyclophosphamide.
C. IV methyl prednisolone.
D. Baclofen tablets.
E. Intravenous immunoglobulins.

83
Respiratory MCQs
1. which of the following causes of apical lung fibrosis
A. ankylosing spondylitis
B. cryptogenic fibrosing alveolitis
C. rheumatoid arthritis
D. systemic lupus erythematosus
2. regarding pleural effusion which of the following is true
A. heart failure accounts for 5% of transudate effusion
B. plural fluid for acid fast bacilli is highly positive
C. bacterial pneumonia and cancer are the most common causes of
transudate effusion
D. recurrent malignant effusions are treated with prolonged course of
antibiotics
E. ultrasound is a sensitive tool for diagnosis of pleural effusions
3. the followings are indicators of severity of community acquired
pneumonia except
A. confusion
B. respiratory rate > 30
C. hypotension
D. thrombocytosis

84
E. age>65
4. A 70 year old man presented with dyspnea, his ABG show hypoxia
and acute respiratory alkalosis what could be the most likely diagnosis
A. emphysema
B . bronchial asthma
C. opiate syndrome
D. kyphoscoliosis
E. hyperventilation syndrome

5. which of the following is true for primary pulmonary tuberculosis


A. tuberculin test become positive after 48 hours of infection
B. milliary tuberculosis is common among middle age group
C. sparing of pericardium is characteristic
D. pleural effusion aspirate is transudative
E. initial immunological response causes hilar lymphadenopathy
6. which one of the following conditions is known to be associated more
with type 1 respiratory failure
A. Gillian barre syndrome
B. drug overdose
C. ankylosing spondylitis
D. pneumonia
9. which of the following is true regarding management of acute severe
asthma
A. chest X-ray should be done prior to starting therapy
B. PaO2 in analysis of blood gases < 60mmHg is an indication for assisted
ventilation
C. normal PaCo2 in blood gas analysis is a good sign
D. peak expiratory flow has no role in assessing severity of attack
10. which of the following is a recognized cause of exudative pleural
effusion
A. cirrhosis
B. nephrotic syndrome

85
C. congestive heart failure
D. pneumonia
11. which of the following is true regarding pulmonary function test
A. total lung capacity is reduced in emphysema
B. FEV1 is reduced in obstructive airway disease
C. residual volume is reduced in chronic bronchitis
D. normal values are independent of age, gender or body built
12. cystic fibrosis has all of the following features except
A. bronchiectasis
B. autosomal recessive disease
C. pancreatic insufficiency
D. finger clubbing
E. low level of sodium & chloride in sweat
13. common respiratory causes of finger clubbing include all the
following except
A. bronchiectasis
B. carcinoma lung
C. idiopathic pulmonary fibrosis
D. chronic bronchitis
14. the following statement is the true about bronchial carcinoma
A. small cell variety accounts for 60% of cases
B. hypercalcemia always indicate metastasis
C. sputum cytology is of no value in the diagnosis
D. CT scan of the chest is indicated for staging
15. Regarding bronchiectasis all of the following are true except
A. There is a dilatation and destruction of air-space distal to the terminal
bronchioles
B. May complicate by severe hemoptysis
E: Is often associated with anaerobic bacteria
D. May associated with dextrocardia

86
E. Proximal bronchiectasis may associated with broncho-pulmonary
aspergillosis.
16. A carbon monoxide ( transfer factor) not reduced in the following
diseases
A. pulmonary fibrosis
B. left ventricular failure
C. Emphysema
D. late stage of extrinsic allergic alveolitis
E. pulmonary hemorrhage
17. Which one of the following is true In Community acquired
Pneumonia in adults
A. The most common causative organism is pseudomonas aeruginosa
B. Streptococcus pneumonia is a recognized cause of atypical pneumonia
C. Serum urea greater than 80 mg/dI is a bad prognostic feature,
D. All patients should be treated in hospital.
E. Aminoglycosides are the first line of treatment in community acquired
pneumonia
18.Which of the following statements is true regarding pulmonary
embolism
A. A positive D dimer test confirms the diagnosis
B. Thrombolytic therapy is indicated in hypotensive patients
C. S wave in lead I, Q wave in lead Ill and T wave inversion in lead III
(S1Q3T3) are specific ECG changes for pulmonary embolism
D. Chest X ray changes are diagnostic for pulmonary embolism
E. Most patients present with pleuritic chest pain
19. Regarding the management of tuberculosis which of the following
statements is CORRECT
A. Monotherapy with Rifampicin can be used in isolated tuberculous
lymphadenitis
B. Sputum remains infectious for at least 7 weeks after initiation of anti-
tuberculous therapy
C. Drug-induced hepatitis is a recognized side effect of both isoniazid
pyrazinamide

87
D. In pulmonary tuberculosis an anti-tuberculous therapy of at least 12 month
are required.
E. Surgery is the treatment of choice in cases of multi-drug resistant
tuberculosis
20. Indications of treatment of sarcoidosis with steroid include all of the
following except:
A. Stage I disease.
B. CNS involvement.
C. Ocular sarcoidosis.
D. Hypercalcemia.
E. Cardiac sarcoidosis.
21. In patient with bronchiectasis which one of the following features
suggest a specific etiology
A. Finger clubbing.
B. Hemoptysis.
C. Peripheral edema.
D. Blood eosinophilia.
E. Mucopurulent sputum.
22. Causes of hypercapnia include all of the following except:
A. Myasthenia gravis.
B. Central sleep apnea.
C. Ankylosing spondylitis.
D. Acute pneumonia.
E. Motor neuron disease.
23. A severe asthmatic attack is suggested by the followings except:
A. respiratory rate of more than 25 per minute.
B. Loud expiratory and inspiratory wheezes.
C. Pulsus paradoxus.
D. A peak flow of less than 50% of predicted.
E. Arterial PaCO2 of more than 50 mmHg.
24. Which one is Incorrect regarding the complications of
Bronchiectasis

88
A. Empyema
B. Lung collapse
C. Respiratory failure
D. Amyloidosis
E. A and C are incorrect

Respiratory cases

89
1.A 25-year-old man suffer from dyspnea grade IV, chest examination
revealed reduced chest movement on left side. Percussion note is stony dull
on the left side. By auscultation absent breath sound on the left side
What is most appropriate management
A. pleurodesis
B. pleurocentesis
C. chest tube with under water seal
D.NSAIDs
E. intravenous diuretics
2 .A 30-year-old man presented with a 2 month history of productive cough ,
anorexia and weight loss. He had low grade fever in the evening with profuse
sweating, he had type I diabetes with last HbA1C9.5%. chest examination
bronchial breathing and crepitation at left upper lobe
What is the most likely diagnosis
A. bronchiectasis
b. pneumococcal pneumonia
C. pulmonary tuberculosis
D. lung fibrosis
3. A57-year-old male patient being evaluated for dyspnea. On examination
the patient is cachectic, pursed lips. Chest examination increase AP diameter
of the chest, decrease chest expansion bilaterally. By auscultation marked
diminished breath sound all over the chest, bilateral rhonchi
What is the most likely diagnosis
A. Idiopathic pulmonary fibrosis
B. Chronic obstructive pulmonary disease
C. Extrinsic allergic alveolitis
D. Bronchiectasis
4. A 50-year-old female patient presented with 4 months H/O productive
cough with copious amounts of foul smelling purulent sputum ,dyspnea and
wheezing. Physical finding include pallor, clubbing. Chest examination
crackles at lung base and rhonchi
What is the most appropriate diagnostic study to confirm this disease
A. Arterial blood gas
B. Sputum culture

90
C. Spirometry
D. High resolution CT scan of the chest
E. Chest x-ray
5. A 34-year-old female presents with sudden onset dyspnea, right side
pleuritic chest pain and cough. She had a cesarean section two weeks back.
On examination she was anxious, tachypnic, a febrile, BP 110/70mmHg,
pulse 110Bpm. ECG shows sinus tachycardia, S wave in lead I, Q wave in
lead III and T wave inversion in lead III
What is the most appropriate therapeutic option
A. Six month anti-tuberculous therapy
B. Therapeutic doses of heparin
C. Intravenous ceftriaxone 2 grams a day
D. Intravenous corticosteroid
6. A farmer aged 45 years complained of worsened episodes of dry cough
and dyspnea for the past 3 years, his symptom worse at the late of the day
after his farm work. On examination he was distressed, cyanotic respiratory
rate 22, BP 130/80mmHg. Chest examination fine end inspiratory crackles on
bases
What is the most likely diagnosis
A. Pulmonary TB
B. Bronchial asthma
C. Extrinsic allergic alveolitis
D. Bronchiectasis
7. A 72-year-old retired stone mason under follow up for /silicosis and mild
COPD attends the Emergency department with increasing shortness of
breath, productive cough and wheeze On examination his respiratory rate is
32, oxygen saturations 63% on air, pulse 126 bpm and BP 135/57 mmHg. A
blood gas is performed:PH7.21 (normal value 7.36 - 7.44) PO2 7.5 kPa
(normal value 11.3 - 12.6) PCO2 8.0 kPa (normal value 4.7 - 6.0) bicarbonate
10 mmol/l( normal value20 = 28)
What do the blood gases show
A. Metabolic acidosis.
B. Metabolic acidosis with respiratory compensation.
C. Mixed metabolic and respiratory acidosis.
D. Respiratory acidosis.

91
E. Respiratory alkalosis.
8. A 67-year-old smoker who lives in a men's hostel attends the Emergency
department with back pain on a background of a cough which has been
present for approximately nine months and intermittently has hemoptysis. He
lost a lot of his weight. He smokes 40 cigarettes a day. He also reports urinary
frequency and hesitancy, but no hematuria. He undergoes a number of
investigations and the results are listed below:Hemoglobin 10.4 g/dl (normal
value 13-18) WBC 11.9 x 109/L (normal value 4-11) ) PIt 342 × 109/L (normal
value 150-400) ) PSA 3ug/L (normal value <4) )CXR: cavitating lesions in both
upper lobes, Lumbar spine x ray: suspicious lesions in L3 and L5, Sputum
microscopy: multiple red rods on a blue background. Bone biopsy: caseating
granuloma.
What is the most likely diagnosis
A. Lung cancer.
B. Multiple myeloma.
C. Prostate cancer.
D. Staphylococcus aureus infection.
E. Tuberculosis.
9. A 23-year-old man presents to the Emergency department with a severe
exacerbation of asthma. He is treated with nebulized salbutamol and
ipratropium bromide and 15 L oxygen.
Which is the next step in his management
A. Oral prednisolone.
B. Intravenous hydrocortisone..
C. Intravenous fluids.
D. Intravenous calcium gluconate.
E. Assisted ventilation.
10. A 58-year-old man presents with weight loss and hemoptysis. He has
smoked most of his life. On examination he is (clubbed and has clinical
evidence of right pleural effusion, His serum calcium is 12 mg/dI ( normal
value 8.5- 10.5 mg/dI). A bone scan is normal.
Which of the following histological type of lung cancer is the most likely
to suffer from
A. Adenocarcinoma.
B. Large cell carcinoma.
C. Mesothelioma

92
D. Small cell carcinoma.
E. Squamous cell carcinoma.
11. A 69-year-old retired miner was referred to the outpatient clinic with a two
month history of persistent non-productive cough. He also complained of
exertional dyspnea and left sided chest pain. His appetite had been poor for
the preceding four months and he had lost over 6 kg in weight. He had retired
aged 45 years after working underground for over thirty years. He lived with
his wife and was an ex-smoker of forty cigarettes a day.
On examination he was a pyrexial. Pulse regular at 90 beats per minute,
blood pressure 135/75 mmHg. There was no finger clubbing. Chest expansion
was reduced on the left with dullness to percussion and diminished breath
sounds at the left base and mid-zone. A chest radiograph showed calcified
pleural plaques and a large left pleural effusion.
Which investigation is the best to yield a diagnosis
A. Bronchoscopy.
B. Computed tomography.
C. Lung ultrasound scan
D. Pleural biopsy.
E. Sputum cytology

Cardiovascular MCQs
93
1. Which of the following statements is true regarding pathophysiology
of heart failure
A. Myocardial injury results in reduced baroreceptor stimulation.
B.As part of the compensatory mechanisms there is reduced preload.
C. There is an increase in circulating aldosterone.
D. The various compensatory mechanisms to maintain cardiac output are self-
limiting
E. BNP( B type natriuretic peptide) has a limited role
2. Which of the following statements correlates best with aortic stenosis
A. Syncope carries good prognosis.
B. Narrow pulse pressure and slow rising carotid pulse is present in mild
disease
C. Echocardiography could correctly estimate stenosis severity
D. Pure aortic stenosis is usually rheumatic in origin
E. Is unlikely to be associated with coronary artery disease
3.Based on the recent IE diagnostic criteria, the two most important
parameters for the diagnosis of this infection are
A. Laboratory abnormalities and blood culture
B. Positive blood culture and Echocardiographic changes
C. ECG changes and physical findings
D. Positive physical findings and positive blood culture
E .High ESR and positive blood culture
4.Which of the following is true regarding Hypertrophic Cardiomyopathy
A. It is transmitted as an autosomal recessive trait
B. ECG is usually normal
C. Malignant ventricular arrhythmias are a rare presentation
D. B blockers are used in the treatment
5. Which of the following describes atrial fibrillation
A. Mortality is less in women
B. Most frequently is secondary to valvular heart disease
C. Use of anticoagulants is mandatory post stroke only
D. It is the least common form of arrhythmias

94
E. Is a cardiac manifestation of thyroid disease.
6. which one of the following is false regarding clinical feature of aortic
stenosis
A. rapid rising pulse
B. patient may asymptomatic
C. episode of acute pulmonary edema
D. narrow pulse pressure
7. regarding SVT which of the following is true
A. has a slow onset
B. ECG reveals narrow QRS complex tachycardia
C. it is common in elderly
D. it carries a high mortality rate
E. syncope is the most common presenting factor
8.. which of the following is recognized cause of secondary
hypertension
A. Addison's disease
B. Sheehan's syndrome
C. hypophysitis
D. pheochromocytoma
9. regarding infective endocarditis which of the following is true
A. streptococcus viridans produce a more rapidly progressive infection
B. staphylococcus aureus endocarditis is common among intravenous drug
user
C. transthoracic echo is highly sensitive and specific for diagnosis
D. benzyl penicillin is drug of choice in treatment of acute presentation
10. recognized feature of DCM include all the following except
A. history of alcoholism
B. pan systolic murmur
C. thromboembolic phenomena
D. diastolic dysfunction
11. large volume pulse is expected in all of the following except

95
A. aortic stenosis
B. thyrotoxicosis
C. aortic regurgitation
D. severe anemia
12. the severity of mitral stenosis can be judged by all the following
except
A. length of diastolic murmur
B. proximity of opening snap to the second heart sound
C. degree of left ventricular enlargement
D. mitral valve area
13. cardiac tamponade characteristically associated with all the
following except
A. bradycardia
B. hypotension
C. pulsus paradoxus
D. raised JVP
14. which of the following procedures requires antibiotic prophylaxis
against infective endocarditis in high risk patients
A. colonoscopy
B. dental extraction
C. barium enema
D. upper GlT endoscopy
15. regarding heart failure which of the following improves mortality
A. digoxin
B. loop diuretics
C. calcium channel blockers
D. beta blockers
16. regarding HOCM, which of the following is correct
A. treatment with ACE inhibitors improves mortality
B .patient may present with sudden death
C. it is commonly caused be enterovirus infection

96
D. systolic murmur at tricuspid area is characteristic
17. regarding ventricular tachycardia which of the following is true
A. ECG reveals narrow QRS tachycardia
B. it is recognized complication of acute myocardial infarction
C. it responds well to intravenous digoxin
D. it carries a low mortality rate
18. prolonged central chest pain at rest in high risk patients should
suggest
A. stable angina
B acute coronary syndrome
C. pericarditis
D. acute heart failure
19. in management of chronic heart failure which of the following is true
A. ACE inhibitors reduce mortality and morbidity
B. digoxin should be used in all patients
C. beta blocker only used in severe heart failure
D. diuretics should be up titrated to maximum dose
20..regarding the 2016 modified jones criteria for diagnosing rheumatic
fever which of the following was added as major criteria in high risk
group
A. mono-arthralgia
B. erythema marginatum
C. subclinical carditis
D. chorea
21. most common arrhythmia in MI is
A. ventricular arrhythmia
B. AF
C. SVT
D. Brady arrhythmia
22. AF caused by al the following EXCEPT
A. anxiety

97
B. thyrotoxicosis
C. WPW
D. alcoholic
23. regarding SVT all are true EXCEPT
A. rate between 130-220
B. may be due to RE entry mechanism
C. safely treated with IV lidocaine
D. may be due to WPW
24. features suggestive of V tach all are true except
A. AV dissociation
B. right axis deviation
C. fusion beat
D. capture beat
25. first choice in management of polymorphic V tach. Is
A. amiodarone
B. adenosine
C. sotalol
D. magnesium sulfate
26. which is the first line agent for treatment of symptomatic bradycardia
A. atropine
B. lidocaine
C. epinephrine
D. vasopressin
27. which rhythm is most likely associated with symptomatic
bradycardia
A. V tach.
B. mobitz Il
C. sinus rhythm
D. MAT
28. an adult patient is in cardiac arrest, the monitor shows V fib, you
should immediately

98
A defibrillate at 200
B. intubate the patient
C. start amiodarone
D. start epinephrine
29. which of the following is an expected typical finding in patient with
mitral stenosis
A. early diastolic murmur
B. muffled first heart sound
C. left parasternal heave in pulmonary hypertension
D. opening snap just before 2 heart sound
30. the following feature of AR except
A. wide pulse pressure
B. Austin flint murmur
C. tapping apex beat
D. head nodding
31. which is TRUE about MS
A. the later opening snap the more severe stenosis
B. presystolic accentuation occur in patient with AF
C. graham steel murmur can be heard
D. loud S1 indicate late stage of disease.
32. All of the following are associated with aortic regurgitation except:
A.. Ebstein's anomaly.
B. Marfan's syndrome.
C. Syphilitic aortitis.
D. Ankylosing spondylitis.
E. Takayasu's disease.
33. Which of the followings require antibiotic prophylaxis for dental
procedure
A. Atrial septal defect ASD.
B. History of coronary surgery.
C. Hypertrophic cardiomyopathy.

99
D. Presence of an implanted defibrillator with pericardial leads.
E. None of the above.
34. In rheumatic fever the major criteria for rheumatic fever include all of
the following except:
A. new murmur
B. Swollen knee for the last 7 days.
C. .A geographic shaped skin rash with central pallor on abdomen.
D. Involuntary movement of arm.
E. Fever.
35. The Jerky pulse:-
A. Seen typically in constrictive pericarditis.
B. It is a large volume pulse.
C. Characterized by presence of two domes one in systole and the other in
diastole.
D. The restrictive CMP is an example.
E. Can be associated with sudden cardiac death.

Cardiovascular cases

100
1. -78- year old man came for follow up to cardiac clinic. He is known case of
hypertension and diabetes. He had history of previous myocardial infarction.
he is on metformin tablet, beta blockers, aspirin and statin. By examination :
pulse 70B\M irregularly irregular, blood pressure 120\70 mm Hg. ECG absent
P wave, irregularly irregular rhythm
Which of the following is the most appropriate action
A. synchronized DC shock
B. start oral anticoagulant
C. start clopidogrel
D. start subcutaneous heparin
2. A 25-years- old male patient presented with breathlessness grade III and
fever. He is known case of mitral valve prosthesis for 5 years. By
examination : he is pale, temperature 38.5, BP 120\80 mmHg.
Which of the following is the most important investigation
A. coronary angiography
B. ESR and CRP
C. blood culture and transesophageal echo
D. peripheral blood film
3. A 20-year- old female complains of central chest pain radiating to the
shoulder and increased by deep breathing and movement. There is a history
of upper respiratory tract infection on the last week. ECG: ST elevation with
upward concavity in all leads and PR segment depression
What is the most likely diagnosis
A. acute myocardial infarction
B. musculoskeletal pain
C. pericarditis
D. mediastinitis
4. A 57 years old male known hypertensive for 5 years not taking his
medication for the last 3 months, presented to ER complaining of headaches
and blurring of vision. His blood pressure 240\130 mmHg and has bilateral
papilledema
Which of the following is preferred IV medication
A. glyceryl trinitrate
B. hydralazine
C phentolamin

101
D. sodium nitroprusside
5.A 67 years old male present with 4 hours history of chest pain, ECG shows
ST elevation in lead II, III and aVF
Which of the following test is appropriate
A. serum CK-MB
B. serum LDH
C. serum AST
D. myoglobin
6. 59 years old female presented with a new onset SVT, no history of SOB or
chest pain, no peripheral edema others normal,
the best initial investigation
A. ECG stress test
B. TSH
C. cardiac enzymes
D. echocardiography
7. A 60 year old man who is diabetic and smoker, presented with
diffuse retrosternal crushing chest pain associated with sweating
and vomiting for two hours. His ECG showed ST elevation in lead
-ll, Il, aVF. Next day developed syncopal attack with pulse rate of
35/min. IV atropine was given but patient still not improved.
Which one of the following is the most appropriate next step
A. synchronizes DC shock
B. pacemaker
C. IV amiodarone
D. IV procainamide
8. A 75 year old man complaining of chest pain while climbing
stairs. On examination there is grade 2 murmur that radiates to the
neck. There is a slow rising pulse with low volume
The most likely diagnosis is
A. HOCM
B. AS

102
C. MS
D. PS
9. A 55 year old diabetic male presented to the emergency room
with frequent recurrent retrosternal chest pain at rest.
ECG showed ST depression and t wave inversion
The most likely diagnosis is
A. acute MI
B. stable angina
C. un stable angina
D. GERD
10. Early management should include all the following except
A. admission to CCU
B. Aspirin
C. thrombolytic
D. heparin
11. A 25 year old male patient presented with dyspnea for 2 days.
he had no chronic illness before. He had an episode of fever and
sore throat one week back. O/E he has tachypnea, HR 120/min
CVS: S3 gallop and systolic murmur at mitral area
The most likely diagnosis is
A. pulmonary edema
B. mitral regurgitation
C. viral myocarditis
D. pulmonary embolism
12. 34 year old man comes to ER with palpitations for the past 4 hours. He
has no chest pain, dyspnea or dizziness. His medical history significant with
WPW syndrome with 3 prior episode of SVT, ECG shows AF with rate of
160/min.
what is the best next step in management
A. adenosine
B. digoxin

103
C. procainamide
C. verapamil
13. A 64 year old man with history of HTN and DM comes to the office due to
palpitation for the past 2 weeks, ECG shows AF with rate of 140/min.
the most appropriate next step is
A. adenosine
B. cardioversion
C. lidocaine
D. diltiazem
14. A 37-year- old woman comes to the emergency department due to left
sided weakness that started several hours ago. She has not had any fever,
headache, or vision changes. Over the past 6 months, the patient has had
progressive dyspnea, nocturnal cough, and occasional hemoptysis. She also
has had frequent episodes of palpitations
Which of the following is the most likely diagnosis for this patient
A. Aortic stenosis
B. Hypertrophic cardiomyopathy
C. Mitral stenosis
D. Pulmonary hypertension
15. A 20-year- old woman comes to the office with intermittent chest pain for
the past 3 weeks, She describes the pain as sharp, located to the left to the
sternum and lasting 5_10 second associated with dyspnea. Cardiac
auscultation reveals a systolic murmur at the apex.
Which of the following is the most likely diagnosis
A. bicuspid aortic valve
B, infective endocarditis
C. mitral valve prolapse
D. ventricular septal defect
16. A 70- year ad man is brought to the emergency room because he lost
consciousness while working in the garden . He says that he had severe
episodes of near syncope on exertion recently. He is not taking any
medications. His blood pressure is 110/85 mmHg. cardiac auscultation
reveals election systole murmur with radiation to the carotid arteries. ECG
demonstrate LVH/and ST depression and T wave changes.
Which is the most probable cause of this patient's condition

104
A. rheumatic heart disease
B. bacterial endocarditis
C. congenital anomaly
D. age related sclerotic changes
17. A 36-year-old male with history of intravenous drug abuse, present with 4
Week history of persistent fever, unusual tiredness, dyspnea, and bilateral leg
swelling. Physical examination revealed: temperature 39.5, bilateral pedal
edema, splinter hemorrhage, finger clubbing, and palpable spleen
The most likely diagnosis is
A. alcoholic cardiomyopathy
B. viral myocarditis
C. subacute bacterial endocarditis
D. tuberculous pericarditis
18. which of the following the most important investigation
A. ECG
B. ANP
C. blood culture and echocardiography
D. ESR and CRP
19. A 70 year old man, known case of diabetes presented with sudden onset
grade IV dyspnea and sweating, no chest pain, pulse 120/min regular, BP
110/60 mmHg, JVP not raised, no pedal edema Heart and lung examination
revealed tachycardia with S3, fine bilateral crepitation with few rhonchi.
Which one of the following is the most appropriate initial investigation
A. Chest X-ray
B. Echocardiography
C.Electrocardiogram-ECG
D. Coronary angiography.
E. Arterial blood gases.
20. A 56 years old patient, smoker, not known to suffer from any previous
illnesses, presents to ER with severe retrosternal chest pain of 1 hour
duration associated with profuse sweating BP is 150/90mmHig; pulse was
80/min regular. His initial ECG shows ST elevation in leads V3 to V5 .While

105
you are preparing to initiate his treatment, the patient suddenly collapse, loses
his consciousness & becomes deeply cyanosed.
What is the most likely cause for his collapse
A. Cardiogenic shock.
B. Ventricular fibrillation
C. Myocardial rupture
D. Severe mitral regurgitation
E. Pulmonary thromboembolism.
21. A 28 years old man presents with history of generalized fatigue of several
weeks duration, weight loss and night sweats and one week duration of
dyspnea. He gives history of having valve replacement surgery 2 years ago.
On examination he has a temperature of 37.5° C,pulse rate 110/minute
regular, BP 120/76mmlg, respiratory rate 13/minute. Multiple palpable small
non-tender lymph nodes in the cervical and axillary groups; clubbing in fingers
and toes and tender nodes in the finger pulps. Chest examination normal,
heart sounds: mechanical click first heart sound in apical area, with a systolic
murmur. Abdominal examination reveals non-tender palpable spleen, No
ascites, investigations: ESR-120mm in the first hour. Elevated white blood
cells count. Normal renal function. Transthoracic echocardiography reveals a
mobile echogenic mass attached to the mechanical valve.
Which of the following is correct in the management of this patient
A. Persistent positive blood cultures are an indication for surgery.
B.Vancomycin is the only anti-microbial used for empirical treatment.
C. Treatment is given for two continuous weeks.
D. Successful treatment of abscess formation is possible without surgery , if
anti-microbial therapy is started early.
E. A single blood culture sample is enough to isolate the causative organism.
22. A 54-year-old man presented to emergency department complaining of
epigastric discomfort which began increase after dinner about half an hour.
On examination he is obese , un comfortable restless, his BP is 160/ 98 CVS,
chest and abdominal examination were normal. ECG show elevated ST
segment in anterior chest leads.
Which of the following is the most appropriate nest step in the
management
A. A trial of anti-acid immediately
B. Reassurance and arrangement outpatient Follow up.
C. Arrange for intensive care management

106
D. Begin thrombolytic therapy in the emergency department
E. Arrange for urgent echocardiography
23. A 58-year-old man presents with central crushing chest pain radiating to
his left arm. He has a long history of smoking and takes Ramipril, aspirin and
simvastatin for hypertension and dyslipidemia. A few minutes after admission
he deteriorates, with loss of consciousness. On examination his BP is 80/50
mmHg and the ECG monitor shows he is in ventricular tachycardia. You
arrange to cardio-Vert him, Post-cardioversion his BP recovers to 125/72
mmHg and he receives aspirin and heparin. Investigations: complete blood
count, renal function and electrolytes were normal, Troponin; 5.1(˂0.05)
ECG: Inferior ST elevation, pulse rate 75/min.
Which one of the following represents the most appropriate
management step
A. IV streptokinase.
B. IV diuretics.
C. Percutaneous coronary intervention- PCI.
D. IV nitroglycerine.
E. I adenosine.
24. A 23-year-old homeless patient comes to the Emergency Department. He
has been feeling sick for some considerable time with fevers and rigors and
has now begun to develop a rash. On examination, he is pyrexial 38.1 °C. He
has splinter hemorrhages on examination of his finger nails and a generalized
purpuric rash. Further examination reveals injection sites on both forearms
and evidence that he has tried to inject into his femoral vein. His
investigations revealed anemia with mild leukocytosis, urine blood +.
Which of the following is the most appropriate next investigation
A. ECHO.
B. CT thorax,
C. Urine cultures.
D. Anti -nuclear antibodies.
E. Ultrasound abdomen.
25. A 76-year-old man comes to the emergency unit complaining of
palpitations. Two further episodes occurred during his time in the Emergency
Department, documented as atrial fibrillation with rate of 150 beat per minute.
By the time you examine him his BP is 135/80 mmHg.
Which of the following is the most appropriate treatment for him to
control his ventricular rate

107
A. Beta blockers
B. Amiodarone.
C. Ivabradine.
D. Amlodipine.
E. Adenosine

108
Renal MCQs
1 D 18 D
2 D 19 E
3 C 20 C
4 D 21 A
5 A 22 C
6 D 23 D
7 D 24 D
8 A 25 D
9 D 26 C
10 C 27 C
11 C 28 C
12 E 29 C
13 E 30 C
14 D 31 A
15 B 32 D
16 C 33 E
17 D

Renal cases
1 B 11 C
2 A 12 C
3 C 13 D
4 C 14 B
5 C 15 D
6 A 16 E
7 A 17 A
8 D 18 A
9 D 19 B
10 B 20 B

109
Endocrine MCQs
1 E 15 D
2 E 16 C
3 C 17 C
4 C 18 C
5 B 19 B
6 C 20 B
7 D 21 B
8 D 22 B
9 C 23 E
10 A 24 D
11 D 25 E
12 D 26 B
13 C 27 B
14 C 28 C

Endocrine cases
1 B 12 A
2 B 13 B
3 A 14 B
4 B 15 B
5 B 16 B
6 C 17 D
7 C 18 C
8 E 19 B
9 B 20 D
10 E 21 A
11 A 22 D

110
Rheumatology MCQs
1 B 17 E
2 D 18 A
3 A 19 D
4 D 20 B
5 B 21 D
6 D 22 C
7 D 23 D
8 A 24 D
9 D 25 C
10 B 26 D
11 D 27 B
12 D 28 E
13 B 29 B
14 C 30 E
15 D 31 B
16 C

Rheumatology cases
1 D 8 D
2 B 9 D
3 B 10 C
4 C 11 A
5 B 12 D
6 B 13 C
7 D 14 A

111
Infectious MCQs
1 A 13 D
2 A 14 C
3 E 15 E
4 C 16 B
5 C 17 D
6 A 18 E
7 B 19 B
8 A 20 C
9 D 21 D
10 D 22 D
11 D 23 C
12 C 24 C

Infectious cases
1 B
2 D
3 D
4 C
5 B
6 B
7 D

112
Hematology MCQs
1 D 15 C
2 A 16 E
3 B 17 D
4 E 18 C
5 B 19 B
6 C 20 A
7 C 21 B
8 C 22 D
9 D 23 D
10 C 24 A
11 C 25 A
12 A 26 D
13 C 27 D
14 C 28 B

Hematology cases
1 B 10 C
2 C 11 D
3 B 12 C
4 D 13 D
5 D 14 A
6 A 15 C
7 D 16 C
8 C 17 B
9 A 18 A

113
GIT MCQs
1 A 18 C
2 B 19 A
3 D 20 C
4 A 21 A
5 C 22 E
6 B 23 C
7 B 24 E
8 C 25 B
9 C 26 A
10 C 27 A
11 C 28 C
12 D 29 C
13 C 30 D
14 D 31 D
15 D 3 A
16 C 33 C
17 B 34 B

GIT cases
1 B 11 D
2 C 12 D
3 B 13 B
4 C 14 C
5 D 15 D
6 B 16 B
7 B 17 B
8 C 18 A
9 C 19 C
10 B 20 A

114
CNS MCQs
1 E 10 C
2 D 11 E
3 B 12 D
4 C 13 B
5 D 14 B
6 B 15 C
7 D 16 A
8 B 17 D
9 B 18 C

CNS cases
1 B 10 C
2 C 11 E
3 C 12 C
4 A 13 C
5 A 14 C
6 C 15 C
7 C 16 C
8 B 17 D
9 B 18 A

115
Respiratory MCQs
1 A 12 D
2 E 13 A
3 D 14 E
4 A 15 C
5 E 16 B
6 D 17 C
7 B 18 A
8 D 19 D
9 B 20 D
10 E 21 E
11 D 22 B

Respiratory cases
1 B
2 C
3 B
4 D
5 B
6 C
7 C
8 E
9 B
10 E
11 D

116
Cardiovascular MCQs
1 C 19 A
2 C 20 C
3 B 21 A
4 D 22 A
5 E 23 C
6 A 24 B
7 B 25 D
8 D 26 A
9 B 27 B
10 D 28 A
11 A 29 C
12 C 30 C
13 A 31 C
14 B 32 A
15 D 33 D
16 B 34 E
17 B 35 E
18 B
Cardiovascular cases
1 B 13 D 25-A
2 C 14 C
3 C 15 C
4 D 16 D
5 A 17 C
6 B 18 C
7 B 19 C
8 B 20 B
9 C 21 A
10 C 22 C
11 C 23 C
12 C 24 A
117
118

You might also like